*NURSING > QUESTIONS & ANSWERS > NCLEX-RN Test Bank Quizzes/Exam #1 to #11 Contains 500 Question and Answers. All Answers Provided W (All)

NCLEX-RN Test Bank Quizzes/Exam #1 to #11 Contains 500 Question and Answers. All Answers Provided With Rationale.

Document Content and Description Below

NCLEX-RN Test Bank Practice Quizzes/Exam #1 to #11 1. Which individual is at greatest risk for developing hypertension? A. 45-year-old African American attorney B. 60-year-old Asian American shop... owner C. 40-year-old Caucasian nurse D. 55-year-old Hispanic teacher 2. A child who ingested 15 maximum strength acetaminophen tablets 45 minutes ago is seen in the emergency department. Which of these orders should the nurse do first? A. Gastric lavage PRN B. Acetylcysteine (Mucomyst) for age per pharmacy C. Start an IV Dextrose 5% with 0.33% normal saline to keep vein open D. Activated charcoal per pharmacy 3. Which complication of cardiac catheterization should the nurse monitor for in the initial 24 hours after the procedure? A. angina at rest B. thrombus formation C. dizziness D. falling blood pressure 4. A client is admitted to the emergency room with renal calculi and is complaining of moderate to severe flank pain and nausea. The client’s temperature is 100.8 degrees Fahrenheit. The priority nursing goal for this client is: A. Maintain fluid and electrolyte balance B. Control nausea C. Manage pain D. Prevent urinary tract infection 5. What would the nurse expect to see while assessing the growth of children during their school age years? A. Decreasing amounts of body fat and muscle mass B. Little change in body appearance from year to year C. Progressive height increase of 4 inches each year D. Yearly weight gain of about 5.5 pounds per year 6. At a community health fair, the blood pressure of a 62-year-old client is 160/96 mmHg. The client states “My blood pressure is usually much lower.” The nurse should tell the client to A. go get a blood pressure check within the next 48 to 72 hours B. check blood pressure again in two (2) months C. see the healthcare provider immediately D. visit the health care provider within one (1) week for a BP check 7. The hospital has sounded the call for a disaster drill on the evening shift. Which of these clients would the nurse put first on the list to be discharged in order to make a room available for a new admission? A. A middle-aged client with a history of being ventilator dependent for over seven (7) years and admitted with bacterial pneumonia five days ago. B. A young adult with diabetes mellitus Type 2 for over ten (10) years and admitted with antibiotic-induced diarrhea 24 hours ago. C. An elderly client with a history of hypertension, hypercholesterolemia, and lupus, and was admitted with Stevens-Johnson syndrome that morning. D. An adolescent with a positive HIV test and admitted for acute cellulitis of the lower leg 48 hours ago. 8. A client has been newly diagnosed with hypothyroidism and will take levothyroxine (Synthroid) 50 mcg/day by mouth. As part of the teaching plan, the nurse emphasizes that this medication: A. Should be taken in the morning B. May decrease the client’s energy level C. Must be stored in a dark container D. Will decrease the client’s heart rate 9. A 3-year-old child comes to the pediatric clinic after the sudden onset of findings that include irritability, thick muffled voice, croaking on inspiration, hot to touch, sit leaning forward, tongue protruding, drooling and suprasternal retractions. What should the nurse do first? A. Prepare the child for X-ray of upper airways B. Examine the child’s throat C. Collect a sputum specimen D. Notify the healthcare provider of the child’s status 10. In children suspected to have a diagnosis of diabetes, which one of the following complaints would be most likely to prompt parents to take their school-age child for evaluation? A. Polyphagia B. Dehydration C. Bedwetting D. Weight loss 11. A client comes to the clinic for treatment of recurrent pelvic inflammatory disease. The nurse recognizes that this condition most frequently follows which type of infection? A. Trichomoniasis B. Chlamydia C. Staphylococcus D. Streptococcus 12. An RN who usually works in a spinal rehabilitation unit is floated to the emergency department. Which of these clients should the charge nurse assign to this RN? A. A middle-aged client who says “I took too many diet pills” and “my heart feels like it is racing out of my chest.” B. A young adult who says “I hear songs from heaven. I need money for beer. I quit drinking two (2) days ago for my family. Why are my arms and legs jerking?” C. An adolescent who has been on pain medications terminal cancer with an initial assessment finding pupils and a relaxed respiratory rate of 10, D. An elderly client who reports having taken a “large crack hit” 10 minutes prior to walking into the emergency room. 13. When teaching a client with coronary artery disease about nutrition, the nurse should emphasize A. Eating three (3) balanced meals a day B. Adding complex carbohydrates C. Avoiding very heavy meals D. Limiting sodium to 7 gms per day 14. Which of these findings indicate that a pump to deliver a basal rate of 10 ml per hour plus PRN for pain breakthrough for morphine drip is not working? A. The client complains of discomfort at the IV insertion site B. The client states “I just can’t get relief from my pain.” C. The level of drug is 100 ml at 8 AM and is 80 ml at noon D. The level of the drug is 100 ml at 8 AM and is 50 ml at noon 15. The nurse is speaking at a community meeting about personal responsibility for health promotion. A participant asks about chiropractic treatment for illnesses. What should be the focus of the nurse’s response? A. Electrical energy fields B. Spinal column manipulation C. Mind-body balance D. Exercise of joints 16. The nurse is performing a neurological assessment on a client post right CVA. Which finding, if observed by the nurse, would warrant immediate attention? A. Decrease in level of consciousness B. Loss of bladder control C. Altered sensation to stimuli D. Emotional ability 17. A child who has recently been diagnosed with cystic fibrosis is in a pediatric clinic where a nurse is performing an assessment. Which later finding of this disease would the nurse not expect to see at this time? A. Positive sweat test B. Bulky greasy stools C. Moist, productive cough D. Meconium ileus 18. The home health nurse visits a male client to provide wound care and finds the client lethargic and confused. His wife states he fell down the stairs 2 hours ago. The nurse should A. Place a call to the client’s health care provider for instructions B. Send him to the emergency room for evaluation C. Reassure the client’s wife that the symptoms are transient D. Instruct the client’s wife to call the doctor if his symptoms become worse 19. Which of the following should the nurse implement to prepare a client for a KUB (Kidney, Ureter, Bladder) radiograph test? A. Client must be NPO before the examination B. Enema to be administered prior to the examination C. Medicate client with Lasix 20 mg IV 30 minutes prior to the examination D. No special orders are necessary for this examination 20. The nurse is giving discharge teaching to a client trseven (7) days post myocardial infarction. He asks the nurse why he must wait six (6) weeks before having sexual intercourse. What is the best response by the nurse to this question? A. “You need to regain your strength before attempting such exertion.” B. “When you can climb 2 flights of stairs without problems, it is generally safe.” C. “Have a glass of wine to relax you, then you can try to have sex.” D. “If you can maintain an active walking program, you will have less risk.” 21. A triage nurse has these four (4) clients arrive in the emergency department within 15 minutes. Which client should the triage nurse send back to be seen first? A. A 2-month-old infant with a history of rolling off the bed and has bulging fontanels with crying B. A teenager who got a singed beard while camping C. An elderly client with complaints of frequent liquid brown colored stools D. A middle-aged client with intermittent pain behind the right scapula 22. While planning care for a toddler, the nurse teaches the parents about the expected developmental changes for this age. Which statement by the mother shows that she understands the child’s developmental needs? A. “I want to protect my child from any falls.” B. “I will set limits on exploring the house.” C. “I understand the need to use those new skills.” D. “I intend to keep control over our child.” 23. The nurse is preparing to administer an enteral feeding to a client via a nasogastric feeding tube. The most important action of the nurse is A. Verify correct placement of the tube B. Check that the feeding solution matches the dietary order C. Aspirate abdominal contents to determine the amount of last feeding remaining in stomach D. Ensure that feeding solution is at room temperature 24. The nurse is caring for a client with a serum potassium level of 3.5 mEq/L. The client is placed on a cardiac monitor and receives 40 mEq KCL in 1000 ml of 5% dextrose in water IV. Which of the following EKG patterns indicates to the nurse that the infusions should be discontinued? A. Narrowed QRS complex B. Shortened “PR” interval C. Tall peaked “T” waves D. Prominent “U” waves 25. A nurse prepares to care for a 4-year-old newly admitted for rhabdomyosarcoma. The nurse should alert the staff to pay more attention to the function of which area of the body? A. All striated muscles B. The cerebellum C. The kidneys D. The leg bones 26. The nurse anticipates that for a family who practices Chinese medicine the priority goal would be to: A. Achieve harmony B. Maintain a balance of energy C. Respect life D. Restore yin and yang 27. During an assessment of a client with cardiomyopathy, the nurse finds that the systolic blood pressure has decreased from 145 to 110 mm Hg and the heart rate has risen from 72 to 96 beats per minute and the client complains of periodic dizzy spells. The nurse instructs the client to A. Increase fluids that are high in protein B. Restrict fluids C. Force fluids and reassess blood pressure D. Limit fluids to non-caffeine beverages 28. The nurse prepares the client for insertion of a pulmonary artery catheter (Swan-Ganz catheter). The nurse teaches the client that the catheter will be inserted to provide information about: A. Stroke volume B. Cardiac output C. Venous pressure D. Left ventricular functioning 29. A nurse enters a client’s room to discover that the client has no pulse or respirations. After calling for help, the first action the nurse should take is: A. Start a peripheral IV B. Initiate high-quality chest compressions C. Establish an airway D. Obtain the crash cart 30. A client is receiving digoxin (Lanoxin) 0.25 mg daily. The health care provider has written a new order to give metoprolol (Lopressor) 25 mg B.I.D. In assessing the client prior to administering the medications, which of the following should the nurse report immediately to the health care provider? A. Blood pressure 94/60 B. Heart rate 76 C. Urine output 50 ml/hour D. Respiratory rate 16 31. While assessing a 1-month-old infant, which finding should the nurse report immediately? A. Abdominal respirations B. Irregular breathing rate C. Inspiratory grunt D. Increased heart rate with crying 32. The nurse practicing in a maternity setting recognizes that the postmature fetus is at risk due to A. Excessive fetal weight B. Low blood sugar levels C. Depletion of subcutaneous fat D. Progressive placental insufficiency 33. The nurse is caring for a client who had a total hip replacement four (4) days ago. Which assessment requires the nurse’s immediate attention? A. I have bad muscle spasms in my lower leg of the affected extremity. B. “I just can’t ‘catch my breath’ over the past few minutes and I think I am in grave danger.” C. “I have to use the bedpan to pass my water at least every 1 to 2 hours.” D. “It seems that the pain medication is not working as well today.” 34. A client has been taking furosemide (Lasix) for the past week. The nurse recognizes which finding may indicate the client is experiencing a negative side effect from the medication? A. Weight gain of 5 pounds B. Edema of the ankles C. Gastric irritability D. Decreased appetite 35. A client who is pregnant comes to the clinic for a first visit. The nurse gathers data about her obstetric history, which includes 3-year-old twins at home and a miscarriage 10 years ago at 12 weeks gestation. How would the nurse accurately document this information? A. Gravida 4 para 2 B. Gravida 2 para 1 C. Gravida 3 para 1 D. Gravida 3 para 2 36. The nurse is caring for a client with a venous stasis ulcer. Which nursing intervention would be most effective in promoting healing? A. Apply dressing using sterile technique B. Improve the client’s nutrition status C. Initiate limb compression therapy D. Begin proteolytic debridement 37. A nurse is to administer meperidine hydrochloride (Demerol) 100 mg, atropine sulfate (Atropisol) 0.4 mg, and promethazine hydrochloride (Phenergan) 50 mg IM to a pre-operative client. Which action should the nurse take first? A. Raise the side rails on the bed B. Place the call bell within reach C. Instruct the client to remain in bed D. Have the client empty bladder 38. Which of these statements best describes the characteristics of an effective reward-feedback system? A. Specific feedback is given as close to the event as possible B. Staff is given feedback in equal amounts over time C. Positive statements are to precede a negative statement D. Performance goals should be higher than what is attainable 39. A client with multiple sclerosis plans to begin an exercise program. In addition to discussing the benefits of regular exercise, the nurse should caution the client to avoid activities which A. Increase the heart rate B. Lead to dehydration C. Are considered aerobic D. May be competitive 40. During the evaluation of the quality of home care for a client with Alzheimer’s disease, the priority for the nurse is to reinforce which statement by a family member? A. At least two (2) full meals a day is eaten. B. We go to a group discussion every week at our community center. C. We have safety bars installed in the bathroom and have 24-hour alarms on the doors. D. The medication is not a problem to have it taken three (3) times a day. NCLEX Practice Exam 2 (40 Items) 1. A nurse is reviewing a patient’s medication during shift change. Which of the following medications would be contraindicated if the patient were pregnant? Select all that apply: A. Warfarin (Coumadin) B. Finasteride (Propecia, Proscar) C. Celecoxib (Celebrex) D. Clonidine (Catapres) E. Transdermal nicotine (Habitrol) F. Clofazimine(Lamprene) 2. A nurse is reviewing a patient’s past medical history (PMH). The history indicates the patient has photosensitive reactions to medications. Which of the following drugs is associated with photosensitive reactions? Select all that apply: A. Ciprofloxacin (Cipro) B. Sulfonamide C. Norfloxacin (Noroxin) D. Sulfamethoxazole and Trimethoprim (Bactrim) E. Isotretinoin (Accutane) F. Nitro-Dur patch 3. A patient tells you that her urine is starting to look discolored. If you believe this change is due to medication, which of the following of the patient’s medication does not cause urine discoloration? A. Sulfasalazine B. Levodopa C. Phenolphthalein D. Aspirin 4. You are responsible for reviewing the nursing unit’s refrigerator. Which of the following drug, if found inside the fridge, should be removed? A. Nadolol (Corgard) B. Opened (in-use) Humulin N injection C. Urokinase (Kinlytic) D. Epoetin alfa IV (Epogen) 5. A 34-year-old female has recently been diagnosed with an autoimmune disease. She has also recently discovered that she is pregnant. Which of the following is the only immunoglobulin that will provide protection to the fetus in the womb? A. IgA B. IgD C. IgE D. IgG 6. A second-year nursing student has just suffered a needlestick while working with a patient that is positive for AIDS. Which of the following is the most significant action that nursing student should take? A. Immediately see a social worker. B. Start prophylactic AZT treatment. C. Start prophylactic Pentamidine treatment. D. Seek counseling. 7. A thirty-five-year-old male has been an insulin-dependent diabetic for five years and now is unable to urinate. Which of the following would you most likely suspect? A. Atherosclerosis B. Diabetic nephropathy C. Autonomic neuropathy D. Somatic neuropathy 8. You are taking the history of a 14-year-old girl who has a (BMI) of 18. The girl reports inability to eat, induced vomiting and severe constipation. Which of the following would you most likely suspect? A. Multiple sclerosis B. Anorexia nervosa C. Bulimia nervosa D. Systemic sclerosis 9. A 24-year-old female is admitted to the ER for confusion. This patient has a history of a myeloma diagnosis, constipation, intense abdominal pain, and polyuria. Based on the presenting signs and symptoms, which of the following would you most likely suspect? A. Diverticulosis B. Hypercalcemia C. Hypocalcemia D. Irritable bowel syndrome 10. Rhogam is most often used to treat____ mothers that have a ____ infant. A. RH positive, RH positive B. RH positive, RH negative C. RH negative, RH positive D. RH negative, RH negative 11. A new mother has some questions about phenylketonuria (PKU). Which of the following statements made by a nurse is not correct regarding PKU? A. A Guthrie test can check the necessary lab values. B. The urine has a high concentration of phenylpyruvic acid C. Mental deficits are often present with PKU. D. The effects of PKU are reversible. 12. A patient has taken an overdose of aspirin. Which of the following should a nurse most closely monitor for during acute management of this patient? A. Onset of pulmonary edema B. Metabolic alkalosis C. Respiratory alkalosis D. Parkinson’s disease type symptoms 13. A 50-year-old blind and deaf patient have been admitted to your floor. As the charge nurse, your primary responsibility for this patient is? A. Let others know about the patient’s deficits. B. Communicate with your supervisor your patient safety concerns. C. Continuously update the patient on the social environment. D. Provide a secure environment for the patient. 14. A patient is getting discharged from a skilled nursing facility (SNF). The patient has a history of severe COPD and PVD. The patient is primarily concerned about his ability to breathe easily. Which of the following would be the best instruction for this patient? A. Deep breathing techniques to increase oxygen levels. B. Cough regularly and deeply to clear airway passages. C. Cough following bronchodilator utilization. D. Decrease CO2 levels by increased oxygen take output during meals. 15. A nurse is caring for an infant that has recently been diagnosed with a congenital heart defect. Which of the following clinical signs would most likely be present? A. Slow pulse rate B. Weight gain C. Decreased systolic pressure D. Irregular WBC lab values 16. A mother has recently been informed that her child has Down’s syndrome. You will be assigned to care for the child at shift change. Which of the following characteristics is not associated with Down’s syndrome? A. Simian crease B. Brachycephaly C. Oily skin D. Hypotonicity 17. A client with myocardial infarction is receiving tissue plasminogen activator, alteplase (Activase, tPA). While on the therapy, the nurse plans to prioritize which of the following? A. Observe for neurological changes. B. Monitor for any signs of renal failure. C. Check the food diary. D. Observe for signs of bleeding. 18. A patient asks a nurse, “My doctor recommended I increase my intake of folic acid. What type of foods contain the highest concentration of folic acids?” A. Green vegetables and liver B. Yellow vegetables and red meat C. Carrots D. Milk 19. A nurse is putting together a presentation on meningitis. Which of the following microorganisms has not been linked to meningitis in humans? A. S. pneumoniae B. H. influenzae C. N. meningitidis D. Cl. difficile 20. A nurse is administering blood to a patient who has a low hemoglobin count. The patient asks how long to RBC’s last in my body? The correct response is. A. The life span of RBC is 45 days. B. The life span of RBC is 60 days. C. The life span of RBC is 90 days. D. The life span of RBC is 120 days. 21. A 65-year-old man has been admitted to the hospital for spinal stenosis surgery. When should the discharge training and planning begin for this patient? A. Following surgery B. Upon admit C. Within 48 hours of discharge D. Preoperative discussion 22. A 5-year-old child and has been recently admitted to the hospital. According to Erik Erikson’s psychosocial development stages, the child is in which stage? A. Trust vs. mistrust B. Initiative vs. guilt C. Autonomy vs. shame and doubt D. Intimacy vs. isolation 23. A toddler is 26 months old and has been recently admitted to the hospital. According to Erikson, which of the following stages is the toddler in? A. Trust vs. mistrust B. Initiative vs. guilt C. Autonomy vs. shame and doubt D. Intimacy vs. isolation 24. A young adult is 20 years old and has been recently admitted to the hospital. According to Erikson, which of the following stages is the adult in? A. Trust vs. mistrust B. Initiative vs. guilt C. Autonomy vs. shame D. Intimacy vs. isolation 25. A nurse is making rounds taking vital signs. Which of the following vital signs is abnormal? A. 11-year-old male: 90 BPM, 22 RPM, 100/70 mmHg B. 13-year-old female: 105 BPM, 22 RPM, 105/50 mmHg C. 5-year-old male: 102 BPM, 24 RPM, 90/65 mmHg D. 6-year-old female: 100 BPM, 26 RPM, 90/70 mmHg 26. When you are taking a patient’s history, she tells you she has been depressed and is dealing with an anxiety disorder. Which of the following medications would the patient most likely be taking? A. Amitriptyline (Elavil) B. Calcitonin C. Pergolide mesylate (Permax) D. Verapamil (Calan) 27. Which of the following conditions would a nurse not administer erythromycin? A. Campylobacteriosis infection B. Legionnaire’s disease C. Pneumonia D. Multiple Sclerosis 28. A patient’s chart indicates a history of hyperkalemia. Which of the following would you not expect to see with this patient if this condition were acute? A. Decreased HR B. Paresthesias C. Muscle weakness of the extremities D. Migraines 29. A patient’s chart indicates a history of ketoacidosis. Which of the following would you not expect to see with this patient if this condition were acute? A. Vomiting B. Extreme Thirst C. Weight gain D. Acetone breath smell 30. A patient’s chart indicates a history of meningitis. Which of the following would you not expect to see with this patient if this condition were acute? A. Increased appetite B. Vomiting C. Fever D. Poor tolerance of light 31. A nurse if reviewing a patient’s chart and notices that the patient suffers from conjunctivitis. Which of the following microorganisms is related to this condition? A. Yersinia pestis B. Helicobacter pylori C. Vibrio cholerae D. Haemophilus aegyptius 32. A nurse if reviewing a patient’s chart and notices that the patient suffers from Lyme disease. Which of the following microorganisms is related to this condition? A. Borrelia burgdorferi B. Streptococcus pyogenes C. Bacillus anthracis D. Enterococcus faecalis 33. A fragile 87-year-old female has recently been admitted to the hospital with increased confusion and falls over last two (2) weeks. She is also noted to have a mild left hemiparesis. Which of the following tests is most likely to be performed? A. FBC (full blood count) B. ECG (electrocardiogram) C. Thyroid function tests D. CT scan 34. An 84-year-old male has been losing mobility and gaining weight over the last two (2) months. The patient also has the heater running in his house 24 hours a day, even on warm days. Which of the following tests is most likely to be performed? A. FBC (full blood count) B. ECG (electrocardiogram) C. Thyroid function tests D. CT scan 35. A 20-year-old female attending college is found unconscious in her dorm room. She has a fever and a noticeable rash. She has just been admitted to the hospital. Which of the following tests is most likely to be performed first? A. Blood sugar check B. CT scan C. Blood cultures D. Arterial blood gases 36. A 28-year-old male has been found wandering around in a confused pattern. The male is sweaty and pale. Which of the following tests is most likely to be performed first? A. Blood sugar check B. CT scan C. Blood cultures D. Arterial blood gases 37. A mother is inquiring about her child’s ability to potty train. Which of the following factors is the most important aspect of toilet training? A. The age of the child B. The child’s ability to understand instruction. C. The overall mental and physical abilities of the child. D. Frequent attempts with positive reinforcement. 38. A parent calls the pediatric clinic and is frantic about the bottle of cleaning fluid her child drank 20 minutes. Which of the following is the most important instruction the nurse can give the parent? A. This too shall pass. B. Take the child immediately to the ER C. Contact the Poison Control Center quickly D. Give the child syrup of ipecac 39. A nurse is administering a shot of Vitamin K to a 30 day-old infant. Which of the following target areas is the most appropriate? A. Gluteus maximus B. Gluteus minimus C. Vastus lateralis D. Vastus medialis 40. A nurse has just started her rounds delivering medication. A new patient on her rounds is a 4-year-old boy who is non-verbal. This child does not have on any identification. What should the nurse do? A. Contact the provider B. Ask the child to write their name on paper. C. Ask a coworker about the identification of the child. D. Ask the father who is in the room the child’s name. NCLEX Practice Exam 3 (40 Items) 1. A patient is admitted to the hospital with a diagnosis of primary hyperparathyroidism. A nurse checking the patient’s lab results would expect which of the following changes in laboratory findings? A. Elevated serum calcium. B. Low serum parathyroid hormone (PTH). C. Elevated serum vitamin D. D. Low urine calcium. 2. A patient with Addison’s disease asks a nurse for nutrition and diet advice. Which of the following diet modifications is NOT recommended? A. A diet high in grains. B. A diet with adequate caloric intake. C. A high protein diet. D. A restricted sodium diet. 3. A patient with a history of diabetes mellitus is on the second post-operative day following cholecystectomy. She has complained of nausea and isn’t able to eat solid foods. The nurse enters the room to find the patient confused and shaky. Which of the following is the most likely explanation for the patient’s symptoms? A. Anesthesia reaction. B. Hyperglycemia. C. Hypoglycemia. D. Diabetic ketoacidosis. 4. A nurse assigned to the emergency department evaluates a patient who underwent fiberoptic colonoscopy 18 hours previously. The patient reports increasing abdominal pain, fever, and chills. Which of the following conditions poses the most immediate concern? A. Bowel perforation. B. Viral Gastroenteritis. C. Colon cancer. D. Diverticulitis. 5. A patient is admitted to the same day surgery unit for liver biopsy. Which of the following laboratory tests assesses coagulation? A. Partial thromboplastin time. B. Prothrombin time. C. Platelet count. D. Hemoglobin 6. A nurse is assessing a clinic patient with a diagnosis of hepatitis A. Which of the following is the most likely route of transmission? A. Sexual contact with an infected partner. B. Contaminated food. C. Blood transfusion. D. Illegal drug use. 7. A leukemia patient has a relative who wants to donate blood for transfusion. Which of the following donor medical conditions would prevent this? A. A history of hepatitis C five years previously. B. Cholecystitis requiring cholecystectomy one year previously. C. Asymptomatic diverticulosis. D. Crohn’s disease in remission. 8. A physician has diagnosed acute gastritis in a clinic patient. Which of the following medications would be contraindicated for this patient? A. Naproxen sodium (Naprosyn). B. Calcium carbonate. C. Clarithromycin (Biaxin). D. Furosemide (Lasix). 9. The nurse is conducting nutrition counseling for a patient with cholecystitis. Which of the following information is important to communicate? A. The patient must maintain a low-calorie diet. B. The patient must maintain a high protein/low carbohydrate diet. C. The patient should limit sweets and sugary drinks. D. The patient should limit fatty foods. 10. A patient admitted to the hospital with myocardial infarction develops severe pulmonary edema. Which of the following symptoms should the nurse expect the patient to exhibit? A. Slow, deep respirations. B. Stridor. C. Bradycardia. D. Air hunger. 11. A nurse caring for several patients on the cardiac unit is told that one is scheduled for implantation of an automatic internal cardioverter-defibrillator. Which of the following patients is most likely to have this procedure? A. A patient admitted for myocardial infarction without cardiac muscle damage. B. A post-operative coronary bypass patient, recovering on schedule. C. A patient with a history of ventricular tachycardia and syncopal episodes. D. A patient with a history of atrial tachycardia and fatigue. 12. A patient is scheduled for a magnetic resonance imaging (MRI) scan for suspected lung cancer. Which of the following is a contraindication to the study for this patient? A. The patient is allergic to shellfish. B. The patient has a pacemaker. C. The patient suffers from claustrophobia. D. The patient takes anti-psychotic medication. 13. A nurse calls a physician with the concern that a patient has developed a pulmonary embolism. Which of the following symptoms has the nurse most likely observed? A. The patient is somnolent with decreased response to the family. B. The patient suddenly complains of chest pain and shortness of breath. C. The patient has developed a wet cough and the nurse hears crackles on auscultation of the lungs. D. The patient has a fever, chills, and loss of appetite. 14. A patient comes to the emergency department with abdominal pain. Work-up reveals the presence of a rapidly enlarging abdominal aortic aneurysm. Which of the following actions should the nurse expect? A. The patient will be admitted to the medicine unit for observation and medication. B. The patient will be admitted to the day surgery unit for sclerotherapy. C. The patient will be admitted to the surgical unit and resection will be scheduled. D. The patient will be discharged home to follow-up with his cardiologist in 24 hours. 15. A patient with leukemia is receiving chemotherapy that is known to depress bone marrow. A CBC (complete blood count) reveals a platelet count of 25,000/microliter. Which of the following actions related specifically to the platelet count should be included in the nursing care plan? A. Monitor for fever every 4 hours. B. Require visitors to wear respiratory masks and protective clothing. C. Consider transfusion of packed red blood cells. D. Check for signs of bleeding, including examination of urine and stool for blood. 16. A nurse in the emergency department is observing a 4-year-old child for signs of increased intracranial pressure after a fall from a bicycle, resulting in head trauma. Which of the following signs or symptoms would be cause for concern? A. Bulging anterior fontanel. B. Repeated vomiting. C. Signs of sleepiness at 10 PM. D. Inability to read short words from a distance of 18 inches. 17. A nonimmunized child appears at the clinic with a visible rash. Which of the following observations indicates the child may have rubeola (measles)? A. Small blue-white spots are visible on the oral mucosa. B. The rash begins on the trunk and spreads outward. C. There is low-grade fever. D. The lesions have a “teardrop-on-a-rose-petal” appearance. 18. A child is seen in the emergency department for scarlet fever. Which of the following descriptions of scarlet fever is NOT correct? A. Scarlet fever is caused by infection with group A Streptococcus bacteria. B. “Strawberry tongue” is a characteristic sign. C. Petechiae occur on the soft palate. D. The pharynx is red and swollen. 19. A child weighing 30 kg arrives at the clinic with diffuse itching as the result of an allergic reaction to an insect bite. Diphenhydramine (Benadryl) 25 mg 3 times a day is prescribed. The correct pediatric dose is 5 mg/kg/day. Which of the following best describes the prescribed drug dose? A. It is the correct dose. B. The dose is too low. C. The dose is too high. D. The dose should be increased or decreased, depending on the symptoms. 20. The mother of a 2-month-old infant brings the child to the clinic for a well-baby check. She is concerned because she feels only one testis in the scrotal sac. Which of the following statements about the undescended testis is the most accurate? A. Normally, the testes are descended by birth. B. The infant will likely require surgical intervention. C. The infant probably has with only one testis. D. Normally, the testes descend by one year of age. 21. A child is admitted to the hospital with a diagnosis of Wilms tumor, stage II. Which of the following statements most accurately describes this stage? A. The tumor is less than 3 cm. in size and requires no chemotherapy. B. The tumor did not extend beyond the kidney and was completely resected. C. The tumor extended beyond the kidney but was completely resected. D. The tumor has spread into the abdominal cavity and cannot be resected. 22. A teen patient is admitted to the hospital by his physician who suspects a diagnosis of acute glomerulonephritis. Which of the following findings is consistent with this diagnosis? Note: More than one answer may be correct. A. Urine specific gravity of 1.040. B. Urine output of 350 ml in 24 hours. C. Brown (“tea-colored”) urine. D. Generalized edema. 23. Which of the following conditions most commonly causes acute glomerulonephritis? A. A congenital condition leading to renal dysfunction. B. Prior infection with group A Streptococcus within the past 10-14 days. C. Viral infection of the glomeruli. D. Nephrotic syndrome. 24. An infant with hydrocele is seen in the clinic for a follow-up visit at 1 month of age. The scrotum is smaller than it was at birth, but fluid is still visible on illumination. Which of the following actions is the physician likely to recommend? A. Massaging the groin area twice a day until the fluid is gone. B. Referral to a surgeon for repair. C. No treatment is necessary; the fluid is reabsorbing normally. D. Keeping the infant in a flat, supine position until the fluid is gone. 25. A nurse is caring for a patient with peripheral vascular disease (PVD). The patient complains of burning and tingling of the hands and feet and cannot tolerate touch of any kind. Which of the following is the most likely explanation for these symptoms? A. Inadequate tissue perfusion leading to nerve damage. B. Fluid overload leading to compression of nerve tissue. C. Sensation distortion due to psychiatric disturbance. D. Inflammation of the skin on the hands and feet. 26. A patient in the cardiac unit is concerned about the risk factors associated with atherosclerosis. Which of the following are hereditary risk factors for developing atherosclerosis? A. Family history of heart disease. B. Overweight. C. Smoking. D. Age. 27. Claudication is a well-known effect of peripheral vascular disease. Which of the following facts about claudication is correct? Select all that apply: A. It results when oxygen demand is greater than oxygen supply. B. It is characterized by pain that often occurs during rest. C. It is a result of tissue hypoxia. D. It is characterized by cramping and weakness. 28. A nurse is providing discharge information to a patient with peripheral vascular disease. Which of the following information should be included in instructions? A. Walk barefoot whenever possible. B. Use a heating pad to keep feet warm. C. Avoid crossing the legs. D. Use antibacterial ointment to treat skin lesions at risk of infection. 29. A patient who has been diagnosed with vasospastic disorder (Raynaud’s disease) complains of cold and stiffness in the fingers. Which of the following descriptions is most likely to fit the patient? A. An adolescent male. B. An elderly woman. C. A young woman. D. An elderly man. 30. A 23-year-old patient in the 27th week of pregnancy has been hospitalized on complete bed rest for 6 days. She experiences sudden shortness of breath, accompanied by chest pain. Which of the following conditions is the most likely cause of her symptoms? A. Myocardial infarction due to a history of atherosclerosis. B. Pulmonary embolism due to deep vein thrombosis (DVT). C. Anxiety attack due to worries about her baby’s health. D. Congestive heart failure due to fluid overload. 31. Thrombolytic therapy is frequently used in the treatment of suspected stroke. Which of the following is a significant complication associated with thrombolytic therapy? A. Air embolus. B. Cerebral hemorrhage. C. Expansion of the clot. D. Resolution of the clot. 32. An infant is brought to the clinic by his mother, who has noticed that he holds his head in an unusual position and always faces to one side. Which of the following is the most likely explanation? A. Torticollis, with shortening of the sternocleidomastoid muscle. B. Craniosynostosis, with premature closure of the cranial sutures. C. Plagiocephaly, with flattening of one side of the head. D. Hydrocephalus, with increased head size. 33. An adolescent brings a physician’s note to school stating that he is not to participate in sports due to a diagnosis of Osgood-Schlatter disease. Which of the following statements about the disease is correct? A. The condition was caused by the student’s competitive swimming schedule. B. The student will most likely require surgical intervention. C. The student experiences pain in the inferior aspect of the knee. D. The student is trying to avoid participation in physical education. 34. The clinic nurse asks a 13-year-old female to bend forward at the waist with arms hanging freely. Which of the following assessments is the nurse most likely conducting? A. Spinal flexibility. B. Leg length disparity. C. Hypostatic blood pressure. D. Scoliosis. 35. A clinic nurse interviews a parent who is suspected of abusing her child. Which of the following characteristics is the nurse LEAST likely to find in an abusing parent? A. Low self-esteem. B. Unemployment. C. Self-blame for the injury to the child. D. Single status. 36. A nurse is assigned to the pediatric rheumatology clinic and is assessing a child who has just been diagnosed with juvenile idiopathic arthritis. Which of the following statements about the disease is most accurate? A. The child has a poor chance of recovery without joint deformity. B. Most children progress to adult rheumatoid arthritis. C. Nonsteroidal anti-inflammatory drugs are the first choice in treatment. D. Physical activity should be minimized. 37. A child is admitted to the hospital several days after stepping on a sharp object that punctured her athletic shoe and entered the flesh of her foot. The physician is concerned about osteomyelitis and has ordered parenteral antibiotics. Which of the following actions is done immediately before the antibiotic is started? A. The admission orders are written. B. A blood culture is drawn. C. A complete blood count with differential is drawn. D. The parents arrive. 38. A two-year-old child has sustained an injury to the leg and refuses to walk. The nurse in the emergency department documents swelling of the lower affected leg. Which of the following does the nurse suspect is the cause of the child’s symptoms? A. Possible fracture of the tibia. B. Bruising of the gastrocnemius muscle. C. Possible fracture of the radius. D. No anatomic injury, the child wants his mother to carry him. 39. A toddler has recently been diagnosed with cerebral palsy. Which of the following information should the nurse provide to the parents? Note: More than one answer may be correct. A. Regular developmental screening is important to avoid secondary developmental delays. B. Cerebral palsy is caused by injury to the upper motor neurons and results in motor dysfunction, as well as possible ocular and speech difficulties. C. Developmental milestones may be slightly delayed but usually will require no additional intervention. D. Parent support groups are helpful for sharing strategies and managing health care issues. 40. A child has recently been diagnosed with Duchenne’s muscular dystrophy. The parents are receiving genetic counseling prior to planning another pregnancy. Which of the following statements includes the most accurate information? A. Duchenne’s is an X-linked recessive disorder, so daughters have a 50% chance of being carriers and sons a 50% chance of developing the disease. B. Duchenne’s is an X-linked recessive disorder, so both daughters and sons have a 50% chance of developing the disease. C. Each child has a 1 in 4 (25%) chance of developing the disorder. D. Sons only have a 1 in 4 (25%) chance of developing the disorder. NCLEX Practice Exam 4 (50 Items) 1. The primary reason for rapid continuous rewarming of the area affected by frostbite is to: A. Lessen the amount of cellular damage B. Prevent the formation of blisters C. Promote movement D. Prevent pain and discomfort 2. A client recently started on hemodialysis wants to know how the dialysis will take the place of his kidneys. The nurse’s response is based on the knowledge that hemodialysis works by: A. Passing water through a dialyzing membrane B. Eliminating plasma proteins from the blood C. Lowering the pH by removing nonvolatile acids D. Filtering waste through a dialyzing membrane 3. During a home visit, a client with AIDS tells the nurse that he has been exposed to measles. Which action by the nurse is most appropriate? A. Administer an antibiotic B. Contact the physician for an order for immune globulin C. Administer an antiviral D. Tell the client that he should remain in isolation for 2 weeks 4. A client hospitalized with MRSA (methicillin-resistant staph aureus) is placed on contact precautions. Which statement is true regarding precautions for infections spread by contact? A. The client should be placed in a room with negative pressure. B. Infection requires close contact; therefore, the door may remain open. C. Transmission is highly likely, so the client should wear a mask at all times. D. Infection requires skin-to-skin contact and is prevented by hand washing, gloves, and a gown. 5. A client who is admitted with an above-the-knee amputation tells the nurse that his foot hurts and itches. Which response by the nurse indicates an understanding of phantom limb pain? A. “The pain will go away in a few days.” B. “The pain is due to peripheral nervous system interruptions. I will get you some pain medication.” C. “The pain is psychological because your foot is no longer there.” D. “The pain and itching are due to the infection you had before the surgery.” 6. A client with cancer of the pancreas has undergone a Whipple procedure. The nurse is aware that during the Whipple procedure, the doctor will remove the: A. Head of the pancreas B. Proximal third section of the small intestines C. Stomach and duodenum D. Esophagus and jejunum 7. The physician has ordered a minimal-bacteria diet for a client with neutropenia. The client should be taught to AVOID eating: A. Packed fruits B. Salt C. Fresh raw pepper D. Ketchup 8. A client is discharged home with a prescription for Coumadin (sodium warfarin). The client should be instructed to: A. Have a Protime done monthly B. Eat more fruits and vegetables C. Drink more liquids D. Avoid crowds 9. The nurse is assisting the physician with removal of a central venous catheter. To facilitate removal, the nurse should instruct the client to: A. Perform the Valsalva maneuver as the catheter is advanced B. Turn his head to the left side and hyperextend the neck C. Take slow, deep breaths as the catheter is removed D. Turn his head to the right while maintaining a sniffing position 10. A client has an order for streptokinase. Before administering the medication, the nurse should assess the client for: A. Allergies to pineapples and bananas B. A history of streptococcal infections C. Prior therapy with phenytoin D. A history of alcohol abuse 11. The nurse is providing discharge teaching for the client with leukemia. The client should be told to avoid: A. Using oil- or cream-based soaps B. Flossing between the teeth C. The intake of salt D. Using an electric razor 12. The nurse is changing the ties of the client with a tracheotomy. The safest method of changing the tracheotomy ties is to: A. Apply the new tie before removing the old one. B. Have a helper present. C. Hold the tracheotomy with the nondominant hand while removing the old tie. D. Ask the doctor to suture the tracheostomy in place. 13. The nurse is monitoring a client following a lung resection. The hourly output from the chest tube was 300mL. The nurse should give priority to: A. Turning the client to the left side B. Milking the tube to ensure patency C. Slowing the intravenous infusion D. Notifying the physician 14. The infant is admitted to the unit with tetralogy of falot. The nurse would anticipate an order for which medication? A. Digoxin B. Epinephrine C. Aminophylline D. Atropine 15. The nurse is educating the lady’s club in self-breast exam. The nurse is aware that most malignant breast masses occur in the Tail of Spence. On the diagram below, select where the Tail of Spence is. 16. The toddler is admitted with a cardiac anomaly. The nurse is aware that the infant with a ventricular septal defect will: A. Tire easily B. Grow normally C. Need more calories D. Be more susceptible to viral infections 17. The nurse is monitoring a client with a history of stillborn infants. The nurse is aware that a nonstress test can be ordered for this client to: A. Determine lung maturity B. Measure the fetal activity C. Show the effect of contractions on fetal heart rate D. Measure the wellbeing of the fetus 18. The nurse is evaluating the client who was admitted 8 hours ago for induction of labor. The following graph is noted on the monitor. Which action should be taken first by the nurse? A. Instruct the client to push B. Perform a vaginal exam C. Turn off the Pitocin infusion D. Place the client in a semi-Fowler’s position 19. The nurse notes the following on the ECG monitor. The nurse would evaluate the cardiac arrhythmia as: A. Atrial flutter B. A sinus rhythm C. Ventricular tachycardia D. Atrial fibrillation 20. A client with clotting disorder has an order to continue Lovenox (enoxaparin) injections after discharge. The nurse should teach the client that Lovenox injections should: A. Be injected into the deltoid muscle B. Be injected into the abdomen C. Aspirate after the injection D. Clear the air from the syringe before injections 21. The nurse has a preop order to administer Valium (diazepam) 10mg and Phenergan (promethazine) 25mg. The correct method of administering these medications is to: A. Administer the medications together in one syringe B. Administer the medication separately C. Administer the Valium, wait 5 minutes, and then inject the Phenergan D. Question the order because they cannot be given at the same time 22. A client with frequent urinary tract infections asks the nurse how she can prevent the reoccurrence. The nurse should teach the client to: A. Douche after intercourse B. Void every 3 hours C. Obtain a urinalysis monthly D. Wipe from back to front after voiding 23. Which task should be assigned to the nursing assistant? A. Placing the client in seclusion B. Emptying the Foley catheter of the preeclamptic client C. Feeding the client with dementia D. Ambulating the client with a fractured hip 24. The client has recently returned from having a thyroidectomy. The nurse should keep which of the following at the bedside? A. A tracheotomy set B. A padded tongue blade C. An endotracheal tube D. An airway 25. The physician has ordered a histoplasmosis test for the elderly client. The nurse is aware that histoplasmosis is transmitted to humans by: A. Cats B. Dogs C. Turtles D. Birds 26. What’s the first intervention for a patient experiencing chest pain and an p02 of 89%? A. Administer morphine. B. Administer oxygen. C. Administer sublingual nitroglycerin. D. Obtain an electrocardiogram (ECC) 27. Which of the following signs and symptoms usually signifies rapid expansion and impending rupture of an abdominal aortic aneurysm? A. Abdominal pain. B. Absent pedal pulses. C. Chest pain. D. Lower back pain. 28. In which of the following types of cardiomyopathy does cardiac output remain normal? A. Dilated. B. Hypertrophic. C. Obliterative. D. Restrictive. 29. Which of the following interventions should be your first priority when treating a patient experiencing chest pain while walking? A. Have the patient sit down. B. Get the patient back to bed. C. Obtain an ECG. D. Administer sublingual nitroglycerin. 30. Which of the following positions would best aid breathing for a patient with acute pulmonary edema? A. Lying flat in bed. B. Left side-lying position. C. High Fowler’s position. D. Semi-Fowler’s position. 31. A pregnant woman arrives at the emergency department (ED) with abruptio placentae at 34 weeks’ gestation. She’s at risk for which of the following blood dyscrasias? A. Thrombocytopenia. B. Idiopathic thrombocytopenic purpura (ITP). C. Disseminated intravascular coagulation (DIC). D. Heparin-associated thrombosis and thrombocytopenia (HATT). 32. A 16-year-old patient involved in a motor vehicle accident arrives in the ED unconscious and severely hypotensive. He’s suspected to have several fractures of his pelvis and legs. Which of the following parenteral fluids is the best choice for his current condition? A. Fresh frozen plasma. B. 0.9% sodium chloride solution. C. Lactated Ringer’s solution. D. Packed red blood cells. 33. Corticosteroids are potent suppressors of the body’s inflammatory response. Which of the following conditions or actions do they suppress? A. Cushing syndrome. B. Pain receptors. C. Immune response. D. Neural transmission. 34. A patient infected with human immunodeficiency virus (HIV) begins zidovudine therapy. Which of the following statements best describes this drug’s action? A. It destroys the outer wall of the virus and kills it. B. It interferes with viral replication. C. It stimulates the immune system. D. It promotes excretion of viral antibodies. 35. A 20-year-old patient is being treated for pneumonia. He has a persistent cough and complains of severe pain on coughing. What could you tell him to help him reduce his discomfort? A. “Hold your cough as much as possible.” B. “Place the head of your bed flat to help with coughing.” C. “Restrict fluids to help decrease the amount of sputum.” D. “Splint your chest wall with a pillow for comfort.” 36. A 19-year-old patient comes to the ED with acute asthma. His respiratory rate is 44 breaths/minute, and he appears to be in acute respiratory distress. Which of the following actions should you take first? A. Take a full medical history. B. Give a bronchodilator by nebulizer. C. Apply a cardiac monitor to the patient. D. Provide emotional support for the patient. 37. A firefighter who was involved in extinguishing a house fire is being treated for smoke inhalation. He develops severe hypoxia 48 hours after the incident, requiring intubation and mechanical ventilation. Which of the following conditions has he most likely developed? A. Acute respiratory distress syndrome (ARDS). B. Atelectasis. C. Bronchitis. D. Pneumonia. 38. Which of the following measures best determines that a patient who had a pneumothorax no longer needs a chest tube? A. You see a lot of drainage from the chest tube. B. Arterial blood gas (ABG) levels are normal. C. The chest X-ray continues to show the lung is 35% deflated. D. The water-seal chamber doesn’t fluctuate when no suction is applied. 39. Which of the following nursing interventions should you use to prevent footdrop and contractures in a patient recovering from a subdural hematoma? A. High-top sneakers. B. Low-dose heparin therapy. C. Physical therapy consultation. D. Sequential compressive device. 40. Which of the following signs of increased intracranial pressure (ICP) would appear first after head trauma? A. Bradycardia. B. Large amounts of very dilute urine. C. Restlessness and confusion. D. Widened pulse pressure. 41. When giving intravenous (I.V.) phenytoin, which of the following methods should you use? A. Use an in-line filter. B. Withhold other anticonvulsants. C. Mix the drug with saline solution only. D. Flush the I.V. catheter with dextrose solution. 42. After surgical repair of a hip, which of the following positions is best for the patient’s legs and hips? A. Abduction. B. Adduction. C. Prone. D. Subluxated. 43. Which of the following factors should be the primary focus of nursing management in a patient with acute pancreatitis? A. Nutrition management. B. Fluid and electrolyte balance. C. Management of hypoglycemia. D. Pain control. 44. After a liver biopsy, place the patient in which of the following positions? A. Left side-lying, with the bed flat. B. Right side-lying, with the bed flat. C. Left side-lying, with the bed in semi-Fowler’s position. D. Right side-lying, with the bed in semi-Fowler’s position. 45. Which of the following potentially serious complications could occur with therapy for hypothyroidism? A. Acute hemolytic reaction. B. Angina or cardiac arrhythmia. C. Retinopathy. D. Thrombocytopenia. 46. Adequate fluid replacement and vasopressin replacement are objectives of therapy for which of the following disease processes? A. Diabetes mellitus. B. Diabetes insipidus. C. Diabetic ketoacidosis. D. Syndrome of inappropriate antidiuretic hormone secretion (SIADH). 47. Patients with Type 1 diabetes mellitus may require which of the following changes to their daily routine during periods of infection? A. No changes. B. Less insulin. C. More insulin. D. Oral diabetic agents. 48. On a follow-up visit after having a vaginal hysterectomy, a 32-year-old patient has a decreased hematocrit level. Which of the following complications does this suggest? A. Hematoma. B. Hypovolemia. C. Infection. D. Pulmonary embolus (PE). 49. A patient has partial-thickness burns to both legs and portions of his trunk. Which of the following I.V. fluids is given first? A. Albumin. B. D5W. C. Lactated Ringer’s solution. D. 0.9% sodium chloride solution with 2 mEq of potassium per 100 ml. 50. Which of the following techniques is correct for obtaining a wound culture specimen from a surgical site? A. Thoroughly irrigate the wound before collecting the specimen. B. Use a sterile swab and wipe the crusty area around the outside of the wound. C. Gently roll a sterile swab from the center of the wound outward to collect drainage. D. Use a sterile swab to collect drainage from the dressing. NCLEX Practice Exam 5 (45 Items) 1. A nurse is administering IV furosemide to a patient admitted with congestive heart failure. After the infusion, which of the following symptoms is NOT expected? A. Increased urinary output. B. Decreased edema. C. Decreased pain. D. Decreased blood pressure. 2. There are a number of risk factors associated with coronary artery disease. Which of the following is a modifiable risk factor? A. Obesity. B. Heredity. C. Gender. D. Age. 3. Tissue plasminogen activator (t-PA) is considered for treatment of a patient who arrives in the emergency department following onset of symptoms of myocardial infarction. Which of the following is a contraindication for treatment with t-PA? A. Worsening chest pain that began earlier in the evening. B. History of cerebral hemorrhage. C. History of prior myocardial infarction. D. Hypertension. 4. Following myocardial infarction, a hospitalized patient is encouraged to practice frequent leg exercises and ambulate in the hallway as directed by his physician. Which of the following choices reflects the purpose of exercise for this patient? A. Increases fitness and prevents future heart attacks. B. Prevents bedsores. C. Prevents DVT (deep vein thrombosis). D. Prevent constipations. 5. A patient arrives in the emergency department with symptoms of myocardial infarction, progressing to cardiogenic shock. Which of the following symptoms should the nurse expect the patient to exhibit with cardiogenic shock? A. Hypertension. B. Bradycardia. C. Bounding pulse. D. Confusion. 6. A patient with a history of congestive heart failure arrives at the clinic complaining of dyspnea. Which of the following actions is the first the nurse should perform? A. Ask the patient to lie down on the exam table. B. Draw blood for chemistry panel and arterial blood gas (ABG). C. Send the patient for a chest x-ray. D. Check blood pressure. 7. A clinic patient has recently been prescribed nitroglycerin for treatment of angina. He calls the nurse complaining of frequent headaches. Which of the following responses to the patient is correct? A. “Stop taking the nitroglycerin and see if the headaches improve.” B. “Go to the emergency department to be checked because nitroglycerin can cause bleeding in the brain.” C. “Headaches are a frequent side effect of nitroglycerine because it causes vasodilation.” D. “The headaches are unlikely to be related to the nitroglycerin, so you should see your doctor for further investigation.” 8. A patient received surgery and chemotherapy for colon cancer, completing therapy three (3) months previously, and she is now in remission. At a follow-up appointment, she complains of fatigue following activity and difficulty with concentration at her weekly bridge games. Which of the following explanations could account for her symptoms? A. The symptoms may be the result of anemia caused by chemotherapy. B. The patient may be immunosuppressed. C. The patient may be depressed. D. The patient may be dehydrated. 9. A clinic patient has a hemoglobin concentration of 10.8 g/dL and reports sticking to a strict vegetarian diet. Which of the follow nutritional advice is appropriate? A. The diet is providing adequate sources of iron and requires no changes. B. The patient should add meat to her diet; a vegetarian diet is not advised. C. The patient should use iron cookware to prepare foods, such as dark-green, leafy vegetables and legumes, which are high in iron. D. A cup of coffee or tea should be added to every meal. 10. A hospitalized patient is receiving packed red blood cells (PRBCs) for treatment of severe anemia. Which of the following is the most accurate statement? A. Transfusion reaction is most likely immediately after the infusion is completed. B. PRBCs are best infused slowly through a 20g. IV catheter. C. PRBCs should be flushed with a 5% dextrose solution. D. A nurse should remain in the room during the first 15 minutes of infusion. 11. A patient who has received chemotherapy for cancer treatment is given an injection of Epoetin. Which of the following should reflect the findings in a complete blood count (CBC) drawn several days later? A. An increase in neutrophil count. B. An increase in hematocrit. C. An increase in platelet count. D. An increase in serum iron. 12. A patient is admitted to the hospital with suspected polycythemia vera. Which of the following symptoms is consistent with the diagnosis? Select all that applies. A. Weight loss. B. Increased clotting time. C. Hypertension. D. Headaches. 13. A nurse is caring for a patient with a platelet count of 20,000/microliter. Which of the following is an important intervention? A. Observe for evidence of spontaneous bleeding. B. Limit visitors to family only. C. Give aspirin in case of headaches. D. Impose immune precautions. 14. A nurse in the emergency department assesses a patient who has been taking long-term corticosteroids to treat renal disease. Which of the following is a typical side effect of corticosteroid treatment? Note: More than one answer may be correct. A. Hypertension. B. Cushingoid features. C. Hyponatremia. D. Low serum albumin. 15. A nurse is caring for patients in the oncology unit. Which of the following is the most important nursing action when caring for a neutropenic patient? A. Change the disposable mask immediately after use. B. Change gloves immediately after use. C. Minimize patient contact. D. Minimize conversation with the patient. 16. A patient is undergoing the induction stage of treatment for leukemia. The nurse teaches family members about infectious precautions. Which of the following statements by family members indicates that the family needs more education? A. We will bring in books and magazines for entertainment. B. We will bring in personal care items for comfort. C. We will bring in fresh flowers to brighten the room. D. We will bring in family pictures and get well cards. 17. A nurse is caring for a patient with acute lymphoblastic leukemia (ALL). Which of the following is the most likely age range of the patient? A. 3-10 years. B. 25-35 years. C. 45-55 years. D. over 60 years. 18. A patient is admitted to the oncology unit for diagnosis of suspected Hodgkin’s disease. Which of the following symptoms is typical of Hodgkin’s disease? A. Painful cervical lymph nodes. B. Night sweats and fatigue. C. Nausea and vomiting. D. Weight gain. 19. The Hodgkin’s disease patient described in the question above undergoes a lymph node biopsy for definitive diagnosis. If the diagnosis of Hodgkin’s disease were correct, which of the following cells would the pathologist expect to find? A. Reed-Sternberg cells. B. Lymphoblastic cells. C. Gaucher’s cells. D. Rieder’s cells 20. A patient is about to undergo bone marrow aspiration and biopsy and expresses fear and anxiety about the procedure. Which of the following is the most effective nursing response? A. Warn the patient to stay very still because the smallest movement will increase her pain. B. Encourage the family to stay in the room for the procedure. C. Stay with the patient and focus on slow, deep breathing for relaxation. D. Delay the procedure to allow the patient to deal with her feelings. 21. A 43-year-old African American male is admitted with sickle cell anemia. The nurse plans to assess circulation in the lower extremities every 2 hours. Which of the following outcome criteria would the nurse use? A. Body temperature of 99°F or less B. Toes moved in active range of motion C. Sensation reported when soles of feet are touched D. Capillary refill of < 3 seconds 22. A 30-year-old male from Haiti is brought to the emergency department in sickle cell crisis. What is the best position for this client? A. Side-lying with knees flexed B. Knee-chest C. High Fowler’s with knees flexed D. Semi-Fowler’s with legs extended on the bed 23. A 25-year-old male is admitted in sickle cell crisis. Which of the following interventions would be of highest priority for this client? A. Taking hourly blood pressures with mechanical cuff B. Encouraging fluid intake of at least 200mL per hour C. Position in high Fowler’s with knee gatch raised D. Administering Tylenol as ordered 24. Which of the following foods would the nurse encourage the client in sickle cell crisis to eat? A. Peaches B. Cottage cheese C. Popsicle D. Lima beans 25. A newly admitted client has sickle cell crisis. The nurse is planning care based on assessment of the client. The client is complaining of severe pain in his feet and hands. The pulse oximetry is 92. Which of the following interventions would be implemented first? Assume that there are orders for each intervention. A. Adjust the room temperature B. Give a bolus of IV fluids C. Start O2 D. Administer meperidine (Demerol) 75 mg IV push 26. The nurse is instructing a client with iron-deficiency anemia. Which of the following meal plans would the nurse expect the client to select? A. Roast beef, gelatin salad, green beans, and peach pie B. Chicken salad sandwich, coleslaw, French fries, ice cream C. Egg salad on wheat bread, carrot sticks, lettuce salad, raisin pie D. Pork chop, creamed potatoes, corn, and coconut cake 27. Clients with sickle cell anemia are taught to avoid activities that cause hypoxia and hypoxemia. Which of the following activities would the nurse recommend? A. A family vacation in the Rocky Mountains B. Chaperoning the local boys club on a snow-skiing trip C. Traveling by airplane for business trips D. A bus trip to the Museum of Natural History 28. The nurse is conducting an admission assessment of a client with vitamin B12 deficiency. Which of the following would the nurse include in the physical assessment? A. Palpate the spleen B. Take the blood pressure C. Examine the feet for petechiae D. Examine the tongue 29. An African American female comes to the outpatient clinic. The physician suspects vitamin B12 deficiency anemia. Because jaundice is often a clinical manifestation of this type of anemia, what body part would be the best indicator? A. Conjunctiva of the eye B. Soles of the feet C. Roof of the mouth D. Shins 30. The nurse is conducting a physical assessment on a client with anemia. Which of the following clinical manifestations would be most indicative of the anemia? A. BP 146/88 B. Respirations 28 shallow C. Weight gain of 10 pounds in 6 months D. Pink complexion 31. The nurse is teaching the client with polycythemia vera about prevention of complications of the disease. Which of the following statements by the client indicates a need for further teaching? A. “I will drink 500mL of fluid or less each day.” B. “I will wear support hose when I am up.” C. “I will use an electric razor for shaving.” D. “I will eat foods low in iron.” 32. A 33-year-old male is being evaluated for possible acute leukemia. Which of the following would the nurse inquire about as a part of the assessment? A. The client collects stamps as a hobby. B. The client recently lost his job as a postal worker. C. The client had radiation for treatment of Hodgkin’s disease as a teenager. D. The client’s brother had leukemia as a child. 33. An African American client is admitted with acute leukemia. The nurse is assessing for signs and symptoms of bleeding. Where is the best site for examining for the presence of petechiae? A. The abdomen B. The thorax C. The earlobes D. The soles of the feet 34. A client with acute leukemia is admitted to the oncology unit. Which of the following would be most important for the nurse to inquire? A. “Have you noticed a change in sleeping habits recently?” B. “Have you had a respiratory infection in the last 6 months?” C. “Have you lost weight recently?” D. “Have you noticed changes in your alertness?” 35. Which of the following would be the priority nursing diagnosis for the adult client with acute leukemia? A. Oral mucous membrane, altered related to chemotherapy B. Risk for injury related to thrombocytopenia C. Fatigue related to the disease process D. Interrupted family processes related to life-threatening illness of a family member 36. A 21-year-old male with Hodgkin’s lymphoma is a senior at the local university. He is engaged to be married and is to begin a new job upon graduation. Which of the following diagnoses would be a priority for this client? A. Sexual dysfunction related to radiation therapy B. Anticipatory grieving related to terminal illness C. Tissue integrity related to prolonged bed rest D. Fatigue related to chemotherapy 37. A client has autoimmune thrombocytopenic purpura. To determine the client’s response to treatment, the nurse would monitor: A. Platelet count B. White blood cell count C. Potassium levels D. Partial prothrombin time (PTT) 38. The home health nurse is visiting a client with autoimmune thrombocytopenic purpura (ATP). The client’s platelet count currently is 80, It will be most important to teach the client and family about: A. Bleeding precautions B. Prevention of falls C. Oxygen therapy D. Conservation of energy 39. A client with a pituitary tumor has had a transsphenoidal hypophysectomy. Which of the following interventions would be appropriate for this client? A. Place the client in Trendelenburg position for postural drainage B. Encourage coughing and deep breathing every 2 hours C. Elevate the head of the bed 30° D. Encourage the Valsalva maneuver for bowel movements 40. The client with a history of diabetes insipidus is admitted with polyuria, polydipsia, and mental confusion. The priority intervention for this client is: A. Measure the urinary output B. Check the vital signs C. Encourage increased fluid intake D. Weigh the client 41. A client with hemophilia has a nosebleed. Which nursing action is most appropriate to control the bleeding? A. Place the client in a sitting position with the head hyperextended B. Pack the nares tightly with gauze to apply pressure to the source of bleeding C. Pinch the soft lower part of the nose for a minimum of 5 minutes D. Apply ice packs to the forehead and back of the neck 42. A client has had a unilateral adrenalectomy to remove a tumor. To prevent complications, the most important measurement in the immediate postoperative period for the nurse to take is: A. Blood pressure B. Temperature C. Output D. Specific gravity 43. A client with Addison’s disease has been admitted with a history of nausea and vomiting for the past 3 days. The client is receiving IV glucocorticoids (Solu-Medrol). Which of the following interventions would the nurse implement? A. Glucometer readings as ordered B. Intake/output measurements C. Sodium and potassium levels monitored D. Daily weights 44. A client had a total thyroidectomy yesterday. The client is complaining of tingling around the mouth and in the fingers and toes. What would the nurse’s next action be? A. Obtain a crash cart B. Check the calcium level C. Assess the dressing for drainage D. Assess the blood pressure for hypertension 45. A 32-year-old mother of three is brought to the clinic. Her pulse is 52, there is a weight gain of 30 pounds in 4 months, and the client is wearing two sweaters. The client is diagnosed with hypothyroidism. Which of the following nursing diagnoses is of highest priority? A. Impaired physical mobility related to decreased endurance B. Hypothermia r/t decreased metabolic rate C. Disturbed thought processes r/t interstitial edema D. Decreased cardiac output r/t bradycardia NCLEX Practice Exam 6 (50 Items) 1. The client is having an arteriogram. During the procedure, the client tells the nurse, “I’m feeling really hot.” Which response would be best? A. “You are having an allergic reaction. I will get an order for Benadryl.” B. “That feeling of warmth is normal when the dye is injected.” C. “That feeling of warmth indicates that the clots in the coronary vessels are dissolving.” D. “I will tell your doctor and let him explain to you the reason for the hot feeling that you are experiencing.” 2. The nurse is observing several healthcare workers providing care. Which action by the healthcare worker indicates a need for further teaching? A. The nursing assistant wears gloves while giving the client a bath. B. The nurse wears goggles while drawing blood from the client. C. The doctor washes his hands before examining the client. D. The nurse wears gloves to take the client’s vital signs. 3. The client is having electroconvulsive therapy for treatment of severe depression. Which of the following indicates that the client’s ECT has been effective? A. The client loses consciousness. B. The client vomits. C. The client’s ECG indicates tachycardia. D. The client has a grand mal seizure. 4. The 5-year-old is being tested for enterobiasis (pinworms). To collect a specimen for assessment of pinworms, the nurse should teach the mother to: A. Examine the perianal area with a flashlight 2 or 3 hours after the child is asleep B. Scrape the skin with a piece of cardboard and bring it to the clinic C. Obtain a stool specimen in the afternoon D. Bring a hair sample to the clinic for evaluation 5. The nurse is teaching the mother regarding treatment for enterobiasis. Which instruction should be given regarding the medication? A. Treatment is not recommended for children less than 10 years of age. B. The entire family should be treated. C. Medication therapy will continue for 1 year. D. Intravenous antibiotic therapy will be ordered. 6. The registered nurse is making assignments for the day. Which client should be assigned to the pregnant nurse? A. The client receiving linear accelerator radiation therapy for lung cancer B. The client with a radium implant for cervical cancer C. The client who has just been administered soluble brachytherapy for thyroid cancer D. The client who returned from placement of iridium seeds for prostate cancer 7. The nurse is planning room assignments for the day. Which client should be assigned to a private room if only one is available? A. The client with Cushing’s disease B. The client with diabetes C. The client with acromegaly D. The client with myxedema 8. The nurse caring for a client in the neonatal intensive care unit administers adult-strength Digitalis to the 3-pound infant. As a result of her actions, the baby suffers permanent heart and brain damage. The nurse can be charged with: A. Negligence B. Tort C. Assault D. Malpractice 9. Which assignment should not be performed by the licensed practical nurse? A. Inserting a Foley catheter B. Discontinuing a nasogastric tube C. Obtaining a sputum specimen D. Starting a blood transfusion 10. The client returns to the unit from surgery with a blood pressure of 90/50, pulse 132, and respirations 30. Which action by the nurse should receive priority? A. Continuing to monitor the vital signs B. Contacting the physician C. Asking the client how he feels D. Asking the LPN to continue the post-op care 11. Which nurse should be assigned to care for the postpartum client with preeclampsia? A. The RN with 2 weeks of experience in postpartum B. The RN with 3 years of experience in labor and delivery C. The RN with 10 years of experience in surgery D. The RN with 1 year of experience in the neonatal intensive care unit 12. Which information should be reported to the state Board of Nursing? A. The facility fails to provide literature in both Spanish and English. B. The narcotic count has been incorrect on the unit for the past 3 days. C. The client fails to receive an itemized account of his bills and services received during his hospital stay. D. The nursing assistant assigned to the client with hepatitis fails to feed the client and give the bath. 13. The nurse is suspected of charting medication administration that he did not give. After talking to the nurse, the charge nurse should: A. Call the Board of Nursing B. File a formal reprimand C. Terminate the nurse D. Charge the nurse with a tort 14. The home health nurse is planning for the day’s visits. Which client should be seen first? A. The 78-year-old who had a gastrectomy 3 weeks ago and has a PEG tube B. The 5-month-old discharged 1 week ago with pneumonia who is being treated with amoxicillin liquid suspension C. The 50-year-old with MRSA being treated with Vancomycin via a PICC line D. The 30-year-old with an exacerbation of multiple sclerosis being treated with cortisone via a centrally placed venous catheter 15. The emergency room is flooded with clients injured in a tornado. Which clients can be assigned to share a room in the emergency department during the disaster? A. A schizophrenic client having visual and auditory hallucinations and the client with ulcerative colitis B. The client who is 6 months pregnant with abdominal pain and the client with facial lacerations and a broken arm C. A child whose pupils are fixed and dilated and his parents, and a client with a frontal head injury D. The client who arrives with a large puncture wound to the abdomen and the client with chest pain 16. The nurse is caring for a 6-year-old client admitted with a diagnosis of conjunctivitis. Before administering eye drops, the nurse should recognize that it is essential to consider which of the following? A. The eye should be cleansed with warm water, removing any exudate, before instilling the eyedrops. B. The child should be allowed to instill his own eye drops. C. The mother should be allowed to instill the eyedrops. D. If the eye is clear from any redness or edema, the eyedrops should be held. 17. The nurse is discussing meal planning with the mother of a 2-year-old toddler. Which of the following statements, if made by the mother, would require a need for further instruction? A. “It is okay to give my child white grape juice for breakfast.” B. “My child can have a grilled cheese sandwich for lunch.” C. “We are going on a camping trip this weekend, and I have bought hot dogs to grill for his lunch.” D. “For a snack, my child can have ice cream.” 18. A 2-year-old toddler is admitted to the hospital. Which of the following nursing interventions would you expect? A. Ask the parent/guardian to leave the room when assessments are being performed. B. Ask the parent/guardian to take the child’s favorite blanket home because anything from the outside should not be brought into the hospital. C. Ask the parent/guardian to room-in with the child. D. If the child is screaming, tell him this is inappropriate behavior. 19. Which instruction should be given to the client who is fitted for a behind-the-ear hearing aid? A. Remove the mold and clean every week. B. Store the hearing aid in a warm place. C. Clean the lint from the hearing aid with a toothpick. D. Change the batteries weekly. 20. A priority nursing diagnosis for a child being admitted from surgery following a tonsillectomy is: A. Body image disturbance B. Impaired verbal communication C. Risk for aspiration D. Pain 21. A client with bacterial pneumonia is admitted to the pediatric unit. What would the nurse expect the admitting assessment to reveal? A. High fever B. Nonproductive cough C. Rhinitis D. Vomiting and diarrhea 22. The nurse is caring for a client admitted with epiglottitis. Because of the possibility of complete obstruction of the airway, which of the following should the nurse have available? A. Intravenous access supplies B. A tracheostomy set C. Intravenous fluid administration pump D. Supplemental oxygen 23. A 25-year-old client with Grave’s disease is admitted to the unit. What would the nurse expect the admitting assessment to reveal? A. Bradycardia B. Decreased appetite C. Exophthalmos D. Weight gain 24. The nurse is providing dietary instructions to the mother of an 8-year-old child diagnosed with celiac disease. Which of the following foods, if selected by the mother, would indicate her understanding of the dietary instructions? A. Ham sandwich on whole-wheat toast B. Spaghetti and meatballs C. Hamburger with ketchup D. Cheese omelet 25. The nurse is caring for an 80-year-old with chronic bronchitis. Upon the morning rounds, the nurse finds an O2 sat of 76%. Which of the following actions should the nurse take first? A. Notify the physician B. Recheck the O2 saturation level in 15 minutes C. Apply oxygen by mask D. Assess the pulse 26. A gravida 3 para 0 is admitted to the labor and delivery unit. The doctor performs an amniotomy. Which observation would the nurse be expected to make after the amniotomy? A. Fetal heart tones 160bpm B. A moderate amount of straw-colored fluid C. A small amount of greenish fluid D. A small segment of the umbilical cord 27. The client is admitted to the unit. A vaginal exam reveals that she is 2cm dilated. Which of the following statements would the nurse expect her to make? A. “We have a name picked out for the baby.” B. “I need to push when I have a contraction.” C. “I can’t concentrate if anyone is touching me.” D. “When can I get my epidural?” 28. The client is having fetal heart rates of 90–110 bpm during the contractions. The first action the nurse should take is: A. Reposition the monitor B. Turn the client to her left side C. Ask the client to ambulate D. Prepare the client for delivery 29. In evaluating the effectiveness of IV Pitocin for a client with secondary dystocia, the nurse should expect: A. A painless delivery B. Cervical effacement C. Infrequent contractions D. Progressive cervical dilation 30. A vaginal exam reveals a footling breech presentation. The nurse should take which of the following actions at this time? A. Anticipate the need for a Caesarean section B. Apply the fetal heart monitor C. Place the client in Genupectoral position D. Perform an ultrasound exam 31. A vaginal exam reveals that the cervix is 4cm dilated, with intact membranes and a fetal heart tone rate of 160–170 bpm. The nurse decides to apply an external fetal monitor. The rationale for this implementation is: A. The cervix is closed. B. The membranes are still intact. C. The fetal heart tones are within normal limits. D. The contractions are intense enough for insertion of an internal monitor. 32. The following are all nursing diagnoses appropriate for a gravida 1 para 0 in labor. Which one would be most appropriate for the primigravida as she completes the early phase of labor? A. Impaired gas exchange related to hyperventilation B. Alteration in placental perfusion related to maternal position C. Impaired physical mobility related to fetal-monitoring equipment D. Potential fluid volume deficit related to decreased fluid intake 33. As the client reaches 8 cm dilation, the nurse notes late decelerations on the fetal monitor. The FHR baseline is 165–175 bpm with variability of 0–2bpm. What is the most likely explanation of this pattern? A. The baby is asleep. B. The umbilical cord is compressed. C. There is a vagal response. D. There is uteroplacental insufficiency. 34. The nurse notes variable decelerations on the fetal monitor strip. The most appropriate initial action would be to: A. Notify her doctor B. Start an IV C. Reposition the client D. Readjust the monitor 35. Which of the following is a characteristic of a reassuring fetal heart rate pattern? A. A fetal heart rate of 170–180 bpm B. A baseline variability of 25–35 bpm C. Ominous periodic changes D. Acceleration of FHR with fetal movements 36. The rationale for inserting a French catheter every hour for the client with epidural anesthesia is: A. The bladder fills more rapidly because of the medication used for the epidural. B. Her level of consciousness is such that she is in a trancelike state. C. The sensation of the bladder filling is diminished or lost. D. She is embarrassed to ask for the bedpan that frequently. 37. A client in the family planning clinic asks the nurse about the most likely time for her to conceive. The nurse explains that conception is most likely to occur when: A. Estrogen levels are low. B. Luteinizing hormone is high. C. The endometrial lining is thin. D. The progesterone level is low. 38. A client tells the nurse that she plans to use the rhythm method of birth control. The nurse is aware that the success of the rhythm method depends on the: A. Age of the client B. Frequency of intercourse C. Regularity of the menses D. Range of the client’s temperature 39. A client with diabetes asks the nurse for advice regarding methods of birth control. Which method of birth control is most suitable for the client with diabetes? A. Intrauterine device B. Oral contraceptives C. Diaphragm D. Contraceptive sponge 40. The doctor suspects that the client has an ectopic pregnancy. Which symptom is consistent with a diagnosis of ectopic pregnancy? A. Painless vaginal bleeding B. Abdominal cramping C. Throbbing pain in the upper quadrant D. Sudden, stabbing pain in the lower quadrant 41. The nurse is teaching a pregnant client about nutritional needs during pregnancy. Which menu selection will best meet the nutritional needs of the pregnant client? A. Hamburger pattie, green beans, French fries, and iced tea B. Roast beef sandwich, potato chips, baked beans, and cola C. Baked chicken, fruit cup, potato salad, coleslaw, yogurt, and iced tea D. Fish sandwich, gelatin with fruit, and coffee 42. The client with hyperemesis gravidarum is at risk for developing: A. Respiratory alkalosis without dehydration B. Metabolic acidosis with dehydration C. Respiratory acidosis without dehydration D. Metabolic alkalosis with dehydration 43. A client tells the doctor that she is about 20 weeks pregnant. The most definitive sign of pregnancy is: A. Elevated human chorionic gonadotropin B. The presence of fetal heart tones C. Uterine enlargement D. Breast enlargement and tenderness 44. The nurse is caring for a neonate whose mother is diabetic. The nurse will expect the neonate to be: A. Hypoglycemic, small for gestational age B. Hyperglycemic, large for gestational age C. Hypoglycemic, large for gestational age D. Hyperglycemic, small for gestational age 45. Which of the following instructions should be included in the nurse’s teaching regarding oral contraceptives? A. Weight gain should be reported to the physician. B. An alternate method of birth control is needed when taking antibiotics. C. If the client misses one or more pills, two pills should be taken per day for 1 week. D. Changes in the menstrual flow should be reported to the physician. 46. The nurse is discussing breastfeeding with a postpartum client. Breastfeeding is contraindicated in the postpartum client with: A. Diabetes B. Positive HIV C. Hypertension D. Thyroid disease 47. A client is admitted to the labor and delivery unit complaining of vaginal bleeding with very little discomfort. The nurse’s first action should be to: A. Assess the fetal heart tones B. Check for cervical dilation C. Check for firmness of the uterus D. Obtain a detailed history 48. A client telephones the emergency room stating that she thinks that she is in labor. The nurse should tell the client that labor has probably begun when: A. Her contractions are 2 minutes apart. B. She has back pain and a bloody discharge. C. She experiences abdominal pain and frequent urination. D. Her contractions are 5 minutes apart. 49. The nurse is teaching a group of prenatal clients about the effects of cigarette smoke on fetal development. Which characteristic is associated with babies born to mothers who smoked during pregnancy? A. Low birth weight B. Large for gestational age C. Preterm birth, but appropriate size for gestation D. Growth retardation in weight and length 50. The physician has ordered an injection of RhoGam for the postpartum client whose blood type is A negative but whose baby is O positive. To provide postpartum prophylaxis, RhoGam should be administered: A. Within 72 hours of delivery B. Within 1 week of delivery C. Within 2 weeks of delivery D. Within 1 month of delivery NCLEX Practice Exam 7 (25 Items) 1. After the physician performs an amniotomy, the nurse’s first action should be to assess the: A. Degree of cervical dilation B. Fetal heart tones C. Client’s vital signs D. Client’s level of discomfort 2. A client is admitted to the labor and delivery unit. The nurse performs a vaginal exam and determines that the client’s cervix is 5 cm dilated with 75% effacement. Based on the nurse’s assessment the client is in which phase of labor? A. Active B. Latent C. Transition D. Early 3. A newborn with narcotic abstinence syndrome is admitted to the nursery. Nursing care of the newborn should include: A. Teaching the mother to provide tactile stimulation B. Wrapping the newborn snugly in a blanket C. Placing the newborn in the infant seat D. Initiating an early infant-stimulation program 4. A client elects to have epidural anesthesia to relieve the discomfort of labor. Following the initiation of epidural anesthesia, the nurse should give priority to: A. Checking for cervical dilation B. Placing the client in a supine position C. Checking the client’s blood pressure D. Obtaining a fetal heart rate 5. The nurse is aware that the best way to prevent postoperative wound infection in the surgical client is to: A. Administer a prescribed antibiotic B. Wash her hands for 2 minutes before care C. Wear a mask when providing care D. Ask the client to cover her mouth when she coughs 6. The elderly client is admitted to the emergency room. Which symptom is the client with a fractured hip most likely to exhibit? A. Pain B. Disalignment C. Cool extremity D. Absence of pedal pulses 7. The nurse knows that a 60-year-old female client’s susceptibility to osteoporosis is most likely related to: A. Lack of exercise B. Hormonal disturbances C. Lack of calcium D. Genetic predisposition 8. A 2-year-old is admitted for repair of a fractured femur and is placed in Bryant’s traction. Which finding by the nurse indicates that the traction is working properly? A. The infant no longer complains of pain. B. The buttocks are 15° off the bed. C. The legs are suspended in the traction. D. The pins are secured within the pulley. 9. A client with a fractured hip has been placed in Buck’s traction. Which statement is true regarding balanced skeletal traction? Balanced skeletal traction: A. Utilizes a Steinman pin B. Requires that both legs be secured C. Utilizes Kirschner wires D. Is used primarily to heal the fractured hips 10. The client is admitted for an open reduction internal fixation of a fractured hip. Immediately following surgery, the nurse should give priority to assessing the: A. Serum collection (Davol) drain B. Client’s pain C. Nutritional status D. Immobilizer 11. Which statement made by the family member caring for the client with a percutaneous gastrostomy tube indicates an understanding of the nurse’s teaching? A. “I must flush the tube with water after feedings and clamp the tube.” B. “I must check placement four times per day.” C. “I will report to the doctor any signs of indigestion.” D. “If my father is unable to swallow, I will discontinue the feeding and call the clinic.” 12. The nurse is assessing the client with a total knee replacement 2 hours postoperative. Which information requires notification of the doctor? A. Bleeding on the dressing is 3cm in diameter. B. The client has a temperature of 100.6°F (38.1°C). C. The client’s hematocrit is 26%. D. The urinary output has been 60 during the last 2 hours. 13. The nurse is caring for the client with a 5-year-old diagnosis of plumbism. Which information in the health history is most likely related to the development of plumbism? A. The client has traveled out of the country in the last 6 months. B. The client’s parents are skilled stained-glass artists. C. The client lives in a house built in 1 D. The client has several brothers and sisters. 14. A client with a total hip replacement requires special equipment. Which equipment would assist the client with a total hip replacement with activities of daily living? A. High-seat commode B. Recliner C. TENS unit D. Abduction pillow 15. An elderly client with an abdominal surgery is admitted to the unit following surgery. In anticipation of complications of anesthesia and narcotic administration, the nurse should: A. Administer oxygen via nasal cannula B. Have narcan (naloxone) available C. Prepare to administer blood products D. Prepare to do cardio resuscitation 16. Which roommate would be most suitable for the 6-year-old male with a fractured femur in Russell’s traction? A. 16-year-old female with scoliosis B. 12-year-old male with a fractured femur C. 10-year-old male with sarcoma D. 6-year-old male with osteomyelitis 17. A client with osteoarthritis has a prescription for Celebrex (celecoxib). Which instruction should be included in the discharge teaching? A. Take the medication with milk. B. Report chest pain. C. Remain upright after taking for 30 minutes. D. Allow 6 weeks for optimal effects. 18. A client with a fractured tibia has a plaster-of-Paris cast applied to immobilize the fracture. Which action by the nurse indicates an understanding of a plaster-of-Paris cast? The nurse: A. Handles the cast with the fingertips B. Petals the cast C. Dries the cast with a hair dryer D. Allows 24 hours before bearing weight 19. The teenager with a fiberglass cast asks the nurse if it will be okay to allow his friends to autograph his cast. Which response would be best? A. “It will be alright for your friends to autograph the cast.” B. “Because the cast is made of plaster, autographing can weaken the cast.” C. “If they don’t use chalk to autograph, it is okay.” D. “Autographing or writing on the cast in any form will harm the cast.” 20. The nurse is assigned to care for the client with a Steinmann pin. During pin care, she notes that the LPN uses sterile gloves and Q-tips to clean the pin. Which action should the nurse take at this time? A. Assisting the LPN with opening sterile packages and peroxide B. Telling the LPN that clean gloves are allowed C. Telling the LPN that the registered nurse should perform pin care D. Asking the LPN to clean the weights and pulleys with peroxide 21. A child with scoliosis has a spica cast applied. Which action specific to the spica cast should be taken? A. Check the bowel sounds B. Assess the blood pressure C. Offer pain medication D. Check for swelling 22. The client with a cervical fracture is placed in traction. Which type of traction will be utilized at the time of discharge? A. Russell’s traction B. Buck’s traction C. Halo traction D. Crutchfield tong traction 23. A client with a total knee replacement has a CPM (continuous passive motion device) applied during the post-operative period. Which statement made by the nurse indicates an understanding of the CPM machine? A. “Use of the CPM will permit the client to ambulate during the therapy.” B. “The CPM machine controls should be positioned distal to the site.” C. “If the client complains of pain during the therapy, I will turn off the machine and call the doctor.” D. “Use of the CPM machine will alleviate the need for physical therapy after the client is discharged.” 24. A client with a fractured hip is being taught correct use of the walker. The nurse is aware that the correct use of the walker is achieved if the: A. Palms rest lightly on the handles B. Elbows are flexed 0° C. Client walks to the front of the walker D. Client carries the walker 25. When assessing a laboring client, the nurse finds a prolapsed cord. The nurse should: A. Attempt to replace the cord B. Place the client on her left side C. Elevate the client’s hips D. Cover the cord with a dry, sterile gauze 1. After the physician performs an amniotomy, the nurse’s first action should be to assess the: A. Degree of cervical dilation B. Fetal heart tones C. Client’s vital signs D. Client’s level of discomfort 2. A client is admitted to the labor and delivery unit. The nurse performs a vaginal exam and determines that the client’s cervix is 5 cm dilated with 75% effacement. Based on the nurse’s assessment the client is in which phase of labor? A. Active B. Latent C. Transition D. Early 3. A newborn with narcotic abstinence syndrome is admitted to the nursery. Nursing care of the newborn should include: A. Teaching the mother to provide tactile stimulation B. Wrapping the newborn snugly in a blanket C. Placing the newborn in the infant seat D. Initiating an early infant-stimulation program 4. A client elects to have epidural anesthesia to relieve the discomfort of labor. Following the initiation of epidural anesthesia, the nurse should give priority to: A. Checking for cervical dilation B. Placing the client in a supine position C. Checking the client’s blood pressure D. Obtaining a fetal heart rate 5. The nurse is aware that the best way to prevent postoperative wound infection in the surgical client is to: A. Administer a prescribed antibiotic B. Wash her hands for 2 minutes before care C. Wear a mask when providing care D. Ask the client to cover her mouth when she coughs 6. The elderly client is admitted to the emergency room. Which symptom is the client with a fractured hip most likely to exhibit? A. Pain B. Disalignment C. Cool extremity D. Absence of pedal pulses 7. The nurse knows that a 60-year-old female client’s susceptibility to osteoporosis is most likely related to: A. Lack of exercise B. Hormonal disturbances C. Lack of calcium D. Genetic predisposition 8. A 2-year-old is admitted for repair of a fractured femur and is placed in Bryant’s traction. Which finding by the nurse indicates that the traction is working properly? A. The infant no longer complains of pain. B. The buttocks are 15° off the bed. C. The legs are suspended in the traction. D. The pins are secured within the pulley. 9. A client with a fractured hip has been placed in Buck’s traction. Which statement is true regarding balanced skeletal traction? Balanced skeletal traction: A. Utilizes a Steinman pin B. Requires that both legs be secured C. Utilizes Kirschner wires D. Is used primarily to heal the fractured hips 10. The client is admitted for an open reduction internal fixation of a fractured hip. Immediately following surgery, the nurse should give priority to assessing the: A. Serum collection (Davol) drain B. Client’s pain C. Nutritional status D. Immobilizer 11. Which statement made by the family member caring for the client with a percutaneous gastrostomy tube indicates an understanding of the nurse’s teaching? A. “I must flush the tube with water after feedings and clamp the tube.” B. “I must check placement four times per day.” C. “I will report to the doctor any signs of indigestion.” D. “If my father is unable to swallow, I will discontinue the feeding and call the clinic.” 12. The nurse is assessing the client with a total knee replacement 2 hours postoperative. Which information requires notification of the doctor? A. Bleeding on the dressing is 3cm in diameter. B. The client has a temperature of 100.6°F (38.1°C). C. The client’s hematocrit is 26%. D. The urinary output has been 60 during the last 2 hours. 13. The nurse is caring for the client with a 5-year-old diagnosis of plumbism. Which information in the health history is most likely related to the development of plumbism? A. The client has traveled out of the country in the last 6 months. B. The client’s parents are skilled stained-glass artists. C. The client lives in a house built in 1 D. The client has several brothers and sisters. 14. A client with a total hip replacement requires special equipment. Which equipment would assist the client with a total hip replacement with activities of daily living? A. High-seat commode B. Recliner C. TENS unit D. Abduction pillow 15. An elderly client with an abdominal surgery is admitted to the unit following surgery. In anticipation of complications of anesthesia and narcotic administration, the nurse should: A. Administer oxygen via nasal cannula B. Have narcan (naloxone) available C. Prepare to administer blood products D. Prepare to do cardio resuscitation 16. Which roommate would be most suitable for the 6-year-old male with a fractured femur in Russell’s traction? A. 16-year-old female with scoliosis B. 12-year-old male with a fractured femur C. 10-year-old male with sarcoma D. 6-year-old male with osteomyelitis 17. A client with osteoarthritis has a prescription for Celebrex (celecoxib). Which instruction should be included in the discharge teaching? A. Take the medication with milk. B. Report chest pain. C. Remain upright after taking for 30 minutes. D. Allow 6 weeks for optimal effects. 18. A client with a fractured tibia has a plaster-of-Paris cast applied to immobilize the fracture. Which action by the nurse indicates an understanding of a plaster-of-Paris cast? The nurse: A. Handles the cast with the fingertips B. Petals the cast C. Dries the cast with a hair dryer D. Allows 24 hours before bearing weight 19. The teenager with a fiberglass cast asks the nurse if it will be okay to allow his friends to autograph his cast. Which response would be best? A. “It will be alright for your friends to autograph the cast.” B. “Because the cast is made of plaster, autographing can weaken the cast.” C. “If they don’t use chalk to autograph, it is okay.” D. “Autographing or writing on the cast in any form will harm the cast.” 20. The nurse is assigned to care for the client with a Steinmann pin. During pin care, she notes that the LPN uses sterile gloves and Q-tips to clean the pin. Which action should the nurse take at this time? A. Assisting the LPN with opening sterile packages and peroxide B. Telling the LPN that clean gloves are allowed C. Telling the LPN that the registered nurse should perform pin care D. Asking the LPN to clean the weights and pulleys with peroxide 21. A child with scoliosis has a spica cast applied. Which action specific to the spica cast should be taken? A. Check the bowel sounds B. Assess the blood pressure C. Offer pain medication D. Check for swelling 22. The client with a cervical fracture is placed in traction. Which type of traction will be utilized at the time of discharge? A. Russell’s traction B. Buck’s traction C. Halo traction D. Crutchfield tong traction 23. A client with a total knee replacement has a CPM (continuous passive motion device) applied during the post-operative period. Which statement made by the nurse indicates an understanding of the CPM machine? A. “Use of the CPM will permit the client to ambulate during the therapy.” B. “The CPM machine controls should be positioned distal to the site.” C. “If the client complains of pain during the therapy, I will turn off the machine and call the doctor.” D. “Use of the CPM machine will alleviate the need for physical therapy after the client is discharged.” 24. A client with a fractured hip is being taught correct use of the walker. The nurse is aware that the correct use of the walker is achieved if the: A. Palms rest lightly on the handles B. Elbows are flexed 0° C. Client walks to the front of the walker D. Client carries the walker 25. When assessing a laboring client, the nurse finds a prolapsed cord. The nurse should: A. Attempt to replace the cord B. Place the client on her left side C. Elevate the client’s hips D. Cover the cord with a dry, sterile gauze NCLEX Practice Exam 8 (50 Items) 1. The client presents to the clinic with a serum cholesterol of 275 mg/dL and is placed on rosuvastatin (Crestor). Which instruction should be given to the client? A. Report muscle weakness to the physician. B. Allow six months for the drug to take effect. C. Take the medication with fruit juice. D. Ask the doctor to perform a complete blood count before starting the medication. 2. The client is admitted to the hospital with hypertensive crises. Diazoxide (Hyperstat) is ordered. During administration, the nurse should: A. Utilize an infusion pump B. Check the blood glucose level C. Place the client in Trendelenburg position D. Cover the solution with foil 3. The 6-month-old client with a ventral septal defect is receiving Digitalis for regulation of his heart rate. Which finding should be reported to the doctor? A. Blood pressure of 126/80 B. Blood glucose of 110 mg/dL C. Heart rate of 60 bpm D. Respiratory rate of 30 per minute 4. The client admitted with angina is given a prescription for nitroglycerin. The client should be instructed to: A. Replenish his supply every 3 months B. Take one every 15 minutes if pain occurs C. Leave the medication in the brown bottle D. Crush the medication and take with water 5. The client is instructed regarding foods that are low in fat and cholesterol. Which diet selection is lowest in saturated fats? A. Macaroni and cheese B. Shrimp with rice C. Turkey breast D. Spaghetti 6. The client is admitted with left-sided congestive heart failure. In assessing the client for edema, the nurse should check the: A. Feet B. Neck C. Hands D. Sacrum 7. The nurse is checking the client’s central venous pressure. The nurse should place the zero of the manometer at the: A. Phlebostatic axis B. PMI C. Erb’s point D. Tail of Spence 8. The physician orders lisinopril (Zestril) and furosemide (Lasix) to be administered concomitantly to the client with hypertension. The nurse should: A. Question the order B. Administer the medications C. Administer separately D. Contact the pharmacy 9. The best method of evaluating the amount of peripheral edema is: A. Weighing the client daily B. Measuring the extremity C. Measuring the intake and output D. Checking for pitting 10. A client with vaginal cancer is being treated with a radioactive vaginal implant. The client’s husband asks the nurse if he can spend the night with his wife. The nurse should explain that: A. Overnight stays by family members is against hospital policy. B. There is no need for him to stay because staffing is adequate. C. His wife will rest much better knowing that he is at home. D. Visitation is limited to 30 minutes when the implant is in place. 11. The nurse is caring for a client hospitalized with a facial stroke. Which diet selection would be suited to the client? A. Roast beef sandwich, potato chips, pickle spear, iced tea B. Split pea soup, mashed potatoes, pudding, milk C. Tomato soup, cheese toast, Jello, coffee D. Hamburger, baked beans, fruit cup, iced tea 12. The physician has prescribed Novolog insulin for a client with diabetes mellitus. Which statement indicates that the client knows when the peak action of the insulin occurs? A. “I will make sure I eat breakfast within 10 minutes of taking my insulin.” B. “I will need to carry candy or some form of sugar with me all the time.” C. “I will eat a snack around three o’clock each afternoon.” D. “I can save my dessert from supper for a bedtime snack.” 13. The nurse is teaching basic infant care to a group of first-time parents. The nurse should explain that a sponge bath is recommended for the first 2 weeks of life because: A. New parents need time to learn how to hold the baby. B. The umbilical cord needs time to separate. C. Newborn skin is easily traumatized by washing. D. The chance of chilling the baby outweighs the benefits of bathing. 14. A client with leukemia is receiving Trimetrexate. After reviewing the client’s chart, the physician orders Wellcovorin (leucovorin calcium). The rationale for administering leucovorin calcium to a client receiving Trimetrexate is to: A. Treat iron-deficiency anemia caused by chemotherapeutic agents B. Create a synergistic effect that shortens treatment time C. Increase the number of circulating neutrophils D. Reverse drug toxicity and prevent tissue damage 15. A 4-month-old is brought to the well-baby clinic for immunization. In addition to the DPT and polio vaccines, the baby should receive: A. HibTITER B. Mumps vaccine C. Hepatitis B vaccine D. MMR 16. The physician has prescribed Nexium (esomeprazole) for a client with erosive gastritis. The nurse should administer the medication: A. 30 minutes before meals B. With each meal C. In a single dose at bedtime D. 30 minutes after meals 17. A client on the psychiatric unit is in an uncontrolled rage and is threatening other clients and staff. What is the most appropriate action for the nurse to take? A. Call security for assistance and prepare to sedate the client. B. Tell the client to calm down and ask him if he would like to play cards. C. Tell the client that if he continues his behavior he will be punished. D. Leave the client alone until he calms down. 18. When the nurse checks the fundus of a client on the first postpartum day, she notes that the fundus is firm, is at the level of the umbilicus, and is displaced to the right. The next action the nurse should take is to: A. Check the client for bladder distention B. Assess the blood pressure for hypotension C. Determine whether an oxytocic drug was given D. Check for the expulsion of small clots 19. A client is admitted to the hospital with a temperature of 99.8°F, complaints of blood-tinged hemoptysis, fatigue, and night sweats. The client’s symptoms are consistent with a diagnosis of: A. Pneumonia B. Reaction to antiviral medication C. Tuberculosis D. Superinfection due to low CD4 count 20. The client is seen in the clinic for treatment of migraine headaches. The drug Imitrex (sumatriptan succinate) is prescribed for the client. Which of the following in the client’s history should be reported to the doctor? A. Diabetes B. Prinzmetal’s angina C. Cancer D. Cluster headaches 21. The client with suspected meningitis is admitted to the unit. The doctor is performing an assessment to determine meningeal irritation and spinal nerve root inflammation. A positive Kernig’s sign is charted if the nurse notes: A. Pain on flexion of the hip and knee B. Nuchal rigidity on flexion of the neck C. Pain when the head is turned to the left side D. Dizziness when changing positions 22. The client with Alzheimer’s disease is being assisted with activities of daily living when the nurse notes that the client uses her toothbrush to brush her hair. The nurse is aware that the client is exhibiting: A. Agnosia B. Apraxia C. Anomia D. Aphasia 23. The client with dementia is experiencing confusion late in the afternoon and before bedtime. The nurse is aware that the client is experiencing what is known as: A. Chronic fatigue syndrome B. Normal aging C. Sundowning D. Delusions 24. The client with confusion says to the nurse, “I haven’t had anything to eat all day long. When are they going to bring breakfast?” The nurse saw the client in the day room eating breakfast with other clients 30 minutes before this conversation. Which response would be best for the nurse to make? A. “You know you had breakfast 30 minutes ago.” B. “I am so sorry that they didn’t get you breakfast. I’ll report it to the charge nurse.” C. “I’ll get you some juice and toast. Would you like something else?” D. “You will have to wait a while; lunch will be here in a little while.” 25. The doctor has prescribed Exelon (rivastigmine) for the client with Alzheimer’s disease. Which side effect is most often associated with this drug? A. Urinary incontinence B. Headaches C. Confusion D. Nausea 26. A client is admitted to the labor and delivery unit in active labor. During examination, the nurse notes a papular lesion on the perineum. Which initial action is most appropriate? A. Document the finding B. Report the finding to the doctor C. Prepare the client for a C-section D. Continue primary care as prescribed 27. A client with a diagnosis of HPV is at risk for which of the following? A. Hodgkin’s lymphoma B. Cervical cancer C. Multiple myeloma D. Ovarian cancer 28. During the initial interview, the client reports that she has a lesion on the perineum. Further investigation reveals a small blister on the vulva that is painful to touch. The nurse is aware that the most likely source of the lesion is: A. Syphilis B. Herpes C. Gonorrhea D. Condylomata 29. A client visiting a family planning clinic is suspected of having an STI. The best diagnostic test for treponema pallidum is: A. Venereal Disease Research Lab (VDRL) B. Rapid plasma reagin (RPR) C. Florescent treponemal antibody (FTA) D. Thayer-Martin culture (TMC) 30. A 15-year-old primigravida is admitted with a tentative diagnosis of HELLP syndrome. Which laboratory finding is associated with HELLP syndrome? A. Elevated blood glucose B. Elevated platelet count C. Elevated creatinine clearance D. Elevated hepatic enzymes 31. The nurse is assessing the deep tendon reflexes of a client with preeclampsia. Which method is used to elicit the biceps reflex? A. The nurse places her thumb on the muscle inset in the antecubital space and taps the thumb briskly with the reflex hammer. B. The nurse loosely suspends the client’s arm in an open hand while tapping the back of the client’s elbow. C. The nurse instructs the client to dangle her legs as the nurse strikes the area below the patella with the blunt side of the reflex hammer. D. The nurse instructs the client to place her arms loosely at her side as the nurse strikes the muscle insert just above the wrist. 32. A primigravida with diabetes is admitted to the labor and delivery unit at 34 weeks gestation. Which doctor’s order should the nurse question? A. Magnesium sulfate 4gm (25%) IV B. Brethine 10 mcg IV C. Stadol 1 mg IV push every 4 hours as needed prn for pain D. Ancef 2gm IVPB every 6 hours 33. A diabetic multigravida is scheduled for an amniocentesis at 32 weeks gestation to determine the L/S ratio and phosphatidyl glycerol level. The L/S ratio is 1:1 and the presence of phosphatidylglycerol is noted. The nurse’s assessment of this data is: A. The infant is at low risk for congenital anomalies. B. The infant is at high risk for intrauterine growth retardation. C. The infant is at high risk for respiratory distress syndrome. D. The infant is at high risk for birth trauma. 34. Which observation in the newborn of a diabetic mother would require immediate nursing intervention? A. Crying B. Wakefulness C. Jitteriness D. Yawning 35. The nurse caring for a client receiving intravenous magnesium sulfate must closely observe for side effects associated with drug therapy. An expected side effect of magnesium sulfate is: A. Decreased urinary output B. Hypersomnolence C. Absence of knee jerk reflex D. Decreased respiratory rate 36. The client has elected to have epidural anesthesia to relieve labor pain. If the client experiences hypotension, the nurse would: A. Place her in Trendelenburg position B. Decrease the rate of IV infusion C. Administer oxygen per nasal cannula D. Increase the rate of the IV infusion 37. A client has cancer of the pancreas. The nurse should be most concerned about which nursing diagnosis? A. Alteration in nutrition B. Alteration in bowel elimination C. Alteration in skin integrity D. Ineffective individual coping 38. The nurse is caring for a client with ascites. Which is the best method to use for determining early ascites? A. Inspection of the abdomen for enlargement B. Bimanual palpation for hepatomegaly C. Daily measurement of abdominal girth D. Assessment for a fluid wave 39. The client arrives in the emergency department after a motor vehicle accident. Nursing assessment findings include BP 80/34, pulse rate 120, and respirations 20. Which is the client’s most appropriate priority nursing diagnosis? A. Alteration in cerebral tissue perfusion B. Fluid volume deficit C. Ineffective airway clearance D. Alteration in sensory perception 40. The home health nurse is visiting an 18-year-old with osteogenesis imperfecta. Which information obtained on the visit would cause the most concern? The client: A. Likes to play football B. Drinks several carbonated drinks per day C. Has two sisters with sickle cell tract D. Is taking acetaminophen to control pain 41. The nurse working the organ transplant unit is caring for a client with a white blood cell count of During evening visitation, a visitor brings a basket of fruit. What action should the nurse take? A. Allow the client to keep the fruit B. Place the fruit next to the bed for easy access by the client C. Offer to wash the fruit for the client D. Tell the family members to take the fruit home 42. The nurse is caring for the client following a laryngectomy when suddenly the client becomes nonresponsive and pale, with a BP of 90/40 systolic. The initial nurse’s action should be to: A. Place the client in Trendelenburg position B. Increase the infusion of Dextrose in normal saline C. Administer atropine intravenously D. Move the emergency cart to the bedside 43. The client admitted 2 days earlier with a lung resection accidentally pulls out the chest tube. Which action by the nurse indicates understanding of the management of chest tubes? A. Order a chest x-ray B. Reinsert the tube C. Cover the insertion site with a Vaseline gauze D. Call the doctor 44. A client being treated with sodium warfarin has a Protime of 120 seconds. Which intervention would be most important to include in the nursing care plan? A. Assess for signs of abnormal bleeding B. Anticipate an increase in the Coumadin dosage C. Instruct the client regarding the drug therapy D. Increase the frequency of neurological assessments 45. Which selection would provide the most calcium for the client who is 4 months pregnant? A. A granola bar B. A bran muffin C. A cup of yogurt D. A glass of fruit juice 46. The client with preeclampsia is admitted to the unit with an order for magnesium sulfate. Which action by the nurse indicates understanding of the possible side effects of magnesium sulfate? A. The nurse places a sign over the bed not to check blood pressure in the right arm. B. The nurse places a padded tongue blade at the bedside. C. The nurse inserts a Foley catheter. D. The nurse darkens the room. 47. A 6-year-old client is admitted to the unit with a hemoglobin of 6g/dL. The physician has written an order to transfuse 2 units of whole blood. When discussing the treatment, the child’s mother tells the nurse that she does not believe in having blood transfusions and that she will not allow her child to have the treatment. What nursing action is most appropriate? A. Ask the mother to leave while the blood transfusion is in progress B. Encourage the mother to reconsider C. Explain the consequences without treatment D. Notify the physician of the mother’s refusal 48. A client is admitted to the unit 2 hours after an explosion causes burns to the face. The nurse would be most concerned with the client developing which of the following? A. Hypovolemia B. Laryngeal edema C. Hypernatremia D. Hyperkalemia 49. The nurse is evaluating nutritional outcomes for a with anorexia nervosa. Which data best indicates that the plan of care is effective? A. The client selects a balanced diet from the menu. B. The client’s hemoglobin and hematocrit improve. C. The client’s tissue turgor improves. D. The client gains weight. 50. The client is admitted following repair of a fractured tibia and cast application. Which nursing assessment should be reported to the doctor? A. Pain beneath the cast B. Warm toes C. Pedal pulses weak and rapid D. Paresthesia of the toes NCLEX Practice Exam 9 (50 Items) 1. A client with a history of abusing barbiturates abruptly stops taking the medication. The nurse should give priority to assessing the client for: A. Depression and suicidal ideation B. Tachycardia and diarrhea C. Muscle cramping and abdominal pain D. Tachycardia and euphoric mood 2. During the assessment of a laboring client, the nurse notes that the FHT are loudest in the upper-right quadrant. The infant is most likely in which position? A. Right breech presentation B. Right occiput anterior presentation C. Left sacral anterior presentation D. Left occipital transverse presentation 3. The primary physiological alteration in the development of asthma is: A. Bronchiolar inflammation and dyspnea B. Hypersecretion of abnormally viscous mucus C. Infectious processes causing mucosal edema D. Spasm of bronchial smooth muscle 4. A client with mania is unable to finish her dinner. To help her maintain sufficient nourishment, the nurse should: A. Serve high-calorie foods she can carry with her B. Encourage her appetite by sending out for her favorite foods C. Serve her small, attractively arranged portions D. Allow her in the unit kitchen for extra food whenever she pleases 5. To maintain Bryant’s traction, the nurse must make certain that the child’s: A. Hips are resting on the bed, with the legs suspended at a right angle to the bed B. Hips are slightly elevated above the bed and the legs are suspended at a right angle to the bed C. Hips are elevated above the level of the body on a pillow and the legs are suspended parallel to the bed D. Hips and legs are flat on the bed, with the traction positioned at the foot of the bed 6. Which action by the nurse indicates understanding of herpes zoster? A. The nurse covers the lesions with a sterile dressing. B. The nurse wears gloves when providing care. C. The nurse administers a prescribed antibiotic. D. The nurse administers oxygen. 7. The client has an order for a trough to be drawn on the client receiving Vancomycin. The nurse is aware that the nurse should contact the lab for them to collect the blood: A. 15 minutes after the infusion B. 30 minutes before the infusion C. 1 hour after the infusion D. 2 hours after the infusion 8. The client using a diaphragm should be instructed to: A. Refrain from keeping the diaphragm in longer than 4 hours B. Keep the diaphragm in a cool location C. Have the diaphragm resized if she gains 5 pounds D. Have the diaphragm resized if she has any surgery 9. The nurse is providing postpartum teaching for a mother planning to breastfeed her infant. Which of the client’s statements indicates the need for additional teaching? A. “I’m wearing a support bra.” B. “I’m expressing milk from my breast.” C. “I’m drinking four glasses of fluid during a 24-hour period.” D. “While I’m in the shower, I’ll allow the water to run over my breasts.” 10. Damage to the VII cranial nerve results in: A. Facial pain B. Absence of ability to smell C. Absence of eye movement D. Tinnitus 11. A client is receiving Pyridium (phenazopyridine hydrochloride) for a urinary tract infection. The client should be taught that the medication may: A. Cause diarrhea B. Change the color of her urine C. Cause mental confusion D. Cause changes in taste 12. Which of the following tests should be performed before beginning a prescription of Accutane? A. Check the calcium level B. Perform a pregnancy test C. Monitor apical pulse D. Obtain a creatinine level 13. A client with AIDS is taking Zovirax (acyclovir). Which nursing intervention is most critical during the administration of acyclovir? A. Limit the client’s activity B. Encourage a high-carbohydrate diet C. Utilize an incentive spirometer to improve respiratory function D. Encourage fluids 14. A client is admitted for an MRI. The nurse should question the client regarding: A. Pregnancy B. A titanium hip replacement C. Allergies to antibiotics D. Inability to move his feet 15. The nurse is caring for the client receiving Amphotericin B. Which of the following indicates that the client has experienced toxicity to this drug? A. Changes in vision B. Nausea C. Urinary frequency D. Changes in skin color 16. The nurse should visit which of the following clients first? A. The client with diabetes with a blood glucose of 95mg/dL B. The client with hypertension being maintained on Lisinopril C. The client with chest pain and a history of angina D. The client with Raynaud’s disease 17. A client with cystic fibrosis is taking pancreatic enzymes. The nurse should administer this medication: A. Once per day in the morning B. Three times per day with meals C. Once per day at bedtime D. Four times per day 18. Cataracts result in opacity of the crystalline lens. Which of the following best explains the functions of the lens? A. The lens controls stimulation of the retina. B. The lens orchestrates eye movement. C. The lens focuses light rays on the retina. D. The lens magnifies small objects. 19. A client who has glaucoma is to have miotic eye drops instilled in both eyes. The nurse knows that the purpose of the medication is to: A. Anesthetize the cornea B. Dilate the pupils C. Constrict the pupils D. Paralyze the muscles of accommodation 20. A client with a severe corneal ulcer has an order for Gentamycin gtt. q 4 hours and Neomycin 1 gtt q 4 hours. Which of the following schedules should be used when administering the drops? A. Allow 5 minutes between the two medications. B. The medications may be used together. C. The medications should be separated by a cycloplegic drug. D. The medications should not be used in the same client. 21. The client with color blindness will most likely have problems distinguishing which of the following colors? A. Orange B. Violet C. Red D. White 22. The client with a pacemaker should be taught to: A. Report ankle edema B. Check his blood pressure daily C. Refrain from using a microwave oven D. Monitor his pulse rate 23. The client with enuresis is being taught regarding bladder retraining. The nurse should advise the client to refrain from drinking after: A. 1900 B. 1200 C. 1000 D. 0700 24. Which of the following diet instructions should be given to the client with recurring urinary tract infections? A. Increase intake of meats. B. Avoid citrus fruits. C. Perform pericare with hydrogen peroxide. D. Drink a glass of cranberry juice every day. 25. The physician has prescribed NPH insulin for a client with diabetes mellitus. Which statement indicates that the client knows when the peak action of the insulin occurs? A. “I will make sure I eat breakfast within 2 hours of taking my insulin.” B. “I will need to carry candy or some form of sugar with me all the time.” C. “I will eat a snack around three o’clock each afternoon.” D. “I can save my dessert from supper for a bedtime snack.” 26. The nurse is caring for a 30-year-old male admitted with a stab wound. While in the emergency room, a chest tube is inserted. Which of the following explains the primary rationale for insertion of chest tubes? A. The tube will allow for equalization of the lung expansion. B. Chest tubes serve as a method of draining blood and serous fluid and assist in reinflating the lungs. C. Chest tubes relieve pain associated with a collapsed lung. D. Chest tubes assist with cardiac function by stabilizing lung expansion. 27. A client who delivered this morning tells the nurse that she plans to breastfeed her baby. The nurse is aware that successful breastfeeding is most dependent on the: A. Mother’s educational level B. Infant’s birth weight C. Size of the mother’s breast D. Mother’s desire to breastfeed 28. The nurse is monitoring the progress of a client in labor. Which finding should be reported to the physician immediately? A. The presence of scant bloody discharge B. Frequent urination C. The presence of green-tinged amniotic fluid D. Moderate uterine contractions 29. The nurse is measuring the duration of the client’s contractions. Which statement is true regarding the measurement of the duration of contractions? A. Duration is measured by timing from the beginning of one contraction to the beginning of the next contraction. B. Duration is measured by timing from the end of one contraction to the beginning of the next contraction. C. Duration is measured by timing from the beginning of one contraction to the end of the same contraction. D. Duration is measured by timing from the peak of one contraction to the end of the same contraction. 30. The physician has ordered an intravenous infusion of Pitocin for the induction of labor. When caring for the obstetric client receiving intravenous Pitocin, the nurse should monitor for: A. Maternal hypoglycemia B. Fetal bradycardia C. Maternal hyperreflexia D. Fetal movement 31. A client with diabetes visits the prenatal clinic at 28 weeks gestation. Which statement is true regarding insulin needs during pregnancy? A. Insulin requirements moderate as the pregnancy progresses. B. A decreased need for insulin occurs during the second trimester. C. Elevations in human chorionic gonadotrophin decrease the need for insulin. D. Fetal development depends on adequate insulin regulation. 32. A client in the prenatal clinic is assessed to have a blood pressure of 180/96. The nurse should give priority to: A. Providing a calm environment B. Obtaining a diet history C. Administering an analgesic D. Assessing fetal heart tones 33. A primigravida, age 42, is 6 weeks pregnant. Based on the client’s age, her infant is at risk for: A. Down syndrome B. Respiratory distress syndrome C. Turner’s syndrome D. Pathological jaundice 34. A client with a missed abortion at 29 weeks gestation is admitted to the hospital. The client will most likely be treated with: A. Magnesium sulfate B. Calcium gluconate C. Dinoprostone (Prostin E.) D. Bromocriptine (Parlodel) 35. A client with preeclampsia has been receiving an infusion containing magnesium sulfate for a blood pressure that is 160/80; deep tendon reflexes are 1 plus, and the urinary output for the past hour is 100mL. The nurse should: A. Continue the infusion of magnesium sulfate while monitoring the client’s blood pressure B. Stop the infusion of magnesium sulfate and contact the physician C. Slow the infusion rate and turn the client on her left side D. Administer calcium gluconate IV push and continue to monitor the blood pressure 36. Which statement made by the nurse describes the inheritance pattern of autosomal recessive disorders? A. An affected newborn has unaffected parents. B. An affected newborn has one affected parent. C. Affected parents have a one in four chance of passing on the defective gene. D. Affected parents have unaffected children who are carriers. 37. A pregnant client, age 32, asks the nurse why her doctor has recommended a serum alpha fetoprotein. The nurse should explain that the doctor has recommended the test: A. Because it is a state law B. To detect cardiovascular defects C. Because of her age D. To detect neurological defects 38. A client with hypothyroidism asks the nurse if she will still need to take thyroid medication during the pregnancy. The nurse’s response is based on the knowledge that: A. There is no need to take thyroid medication because the fetus’s thyroid produces a thyroid-stimulating hormone. B. Regulation of thyroid medication is more difficult because the thyroid gland increases in size during pregnancy. C. It is more difficult to maintain thyroid regulation during pregnancy due to a slowing of metabolism. D. Fetal growth is arrested if thyroid medication is continued during pregnancy. 39. The nurse is responsible for performing a neonatal assessment on a full-term infant. At 1 minute, the nurse could expect to find: A. An apical pulse of 100 B. An absence of tonus C. Cyanosis of the feet and hands D. Jaundice of the skin and sclera 40. A client with sickle cell anemia is admitted to the labor and delivery unit during the first phase of labor. The nurse should anticipate the client’s need for: A. Supplemental oxygen B. Fluid restriction C. Blood transfusion D. Delivery by Caesarean section 41. A client with diabetes has an order for ultrasonography. Preparation for an ultrasound includes: A. Increasing fluid intake B. Limiting ambulation C. Administering an enema D. Withholding food for 8 hours 42. An infant who weighs 8 pounds at birth would be expected to weigh how many pounds at 1 year? A. 14 pounds B. 16 pounds C. 18 pounds D. 24 pounds 43. A pregnant client with a history of alcohol addiction is scheduled for a nonstress test. The nonstress test: A. Determines the lung maturity of the fetus B. Measures the activity of the fetus C. Shows the effect of contractions on the fetal heart rate D. Measures the neurological well-being of the fetus 44. A full-term male has hypospadias. Which statement describes hypospadias? A. The urethral opening is absent. B. The urethra opens on the dorsal side of the penis. C. The penis is shorter than usual. D. The urethral meatus opens on the underside of the penis. 45. A gravida 3 para 2 is admitted to the labor unit. Vaginal exam reveals that the client’s cervix is 8 cm dilated, with complete effacement. The priority nursing diagnosis at this time is: A. Alteration in coping related to pain B. Potential for injury related to precipitate delivery C. Alteration in elimination related to anesthesia D. Potential for fluid volume deficit related to NPO status 46. The client with varicella will most likely have an order for which category of medication? A. Antibiotics B. Antipyretics C. Antivirals D. Anticoagulants 47. A client is admitted complaining of chest pain. Which of the following drug orders should the nurse question? A. Nitroglycerin B. Ampicillin C. Propranolol D. Verapamil 48. Which of the following instructions should be included in the teaching for the client with rheumatoid arthritis? A. Avoid exercise because it fatigues the joints. B. Take prescribed anti-inflammatory medications with meals. C. Alternate hot and cold packs to affected joints. D. Avoid weight-bearing activity. 49. A client with acute pancreatitis is experiencing severe abdominal pain. Which of the following orders should be questioned by the nurse? A. Meperidine 100 mg IM q 4 hours PRN pain B. Mylanta 30 ccs q 4 hours via NG C. Cimetidine 300 mg PO q.i.d. D. Morphine 8 mg IM q 4 hours PRN pain 50. The client is admitted to the chemical dependence unit with an order for continuous observation. The nurse is aware that the doctor has ordered continuous observation because: A. Hallucinogenic drugs create both stimulant and depressant effects. B. Hallucinogenic drugs induce a state of altered perception. C. Hallucinogenic drugs produce severe respiratory depression. D. Hallucinogenic drugs induce rapid physical dependence. NCLEX Practice Exam 10 (20 Questions) 1. A patient arrives at the emergency department complaining of midsternal chest pain. Which of the following nursing action should take priority? A. A complete history with emphasis on preceding events. B. An electrocardiogram. C. Careful assessment of vital signs. D. Chest exam with auscultation. 2. A patient has been hospitalized with pneumonia and is about to be discharged. A nurse provides discharge instructions to a patient and his family. Which misunderstanding by the family indicates the need for more detailed information? A. The patient may resume normal home activities as tolerated but should avoid physical exertion and get adequate rest. B. The patient should resume a normal diet with emphasis on nutritious, healthy foods. C. The patient may discontinue the prescribed course of oral antibiotics once the symptoms have completely resolved. D. The patient should continue use of the incentive spirometer to keep airways open and free of secretions. 3. A nurse is caring for an elderly Vietnamese patient in the terminal stages of lung cancer. Many family members are in the room around the clock performing unusual rituals and bringing ethnic foods. Which of the following actions should the nurse take? A. Restrict visiting hours and ask the family to limit visitors to two at a time. B. Notify visitors with a sign on the door that the patient is limited to clear fluids only with no solid food allowed. C. If possible, keep the other bed in the room unassigned to provide privacy and comfort to the family. D. Contact the physician to report the unusual rituals and activities. 4. The charge nurse on the cardiac unit is planning assignments for the day. Which of the following is the most appropriate assignment for the float nurse that has been reassigned from labor and delivery? A. A one-week postoperative coronary bypass patient, who is being evaluated for placement of a pacemaker prior to discharge. B. A suspected myocardial infarction patient on telemetry, just admitted from the Emergency Department and scheduled for an angiogram. C. A patient with unstable angina being closely monitored for pain and medication titration. D. A postoperative valve replacement patient who was recently admitted to the unit because all surgical beds were filled. 5. A newly diagnosed 8-year-old child with type I diabetes mellitus and his mother are receiving diabetes education prior to discharge. The physician has prescribed Glucagon for emergency use. The mother asks the purpose of this medication. Which of the following statements by the nurse is correct? A. Glucagon enhances the effect of insulin in case the blood sugar remains high one hour after injection. B. Glucagon treats hypoglycemia resulting from insulin overdose. C. Glucagon treats lipoatrophy from insulin injections. D. Glucagon prolongs the effect of insulin, allowing fewer injections. 6. An infant with congestive heart failure is receiving diuretic therapy at home. Which of the following symptoms would indicate that the dosage may need to be increased? A. Sudden weight gain. B. Decreased blood pressure. C. Slow, shallow breathing. D. Bradycardia. 7. A patient taking Dilantin (phenytoin) for a seizure disorder is experiencing breakthrough seizures. A blood sample is taken to determine the serum drug level. Which of the following would indicate a sub-therapeutic level? A. 15 mcg/mL. B. 4 mcg/mL. C. 10 mcg/dL. D. 5 mcg/dL. 8. A patient arrives at the emergency department complaining of back pain. He reports taking at least 3 acetaminophen tablets every three hours for the past week without relief. Which of the following symptoms suggests acetaminophen toxicity? A. Tinnitus. B. Diarrhea. C. Hypertension. D. Hepatic damage. 9. A nurse is caring for a cancer patient receiving subcutaneous morphine sulfate for pain. Which of the following nursing actions is most important in the care of this patient? A. Monitor urine output. B. Monitor respiratory rate. C. Monitor heart rate. D. Monitor temperature. 10. A patient arrives at the emergency department with severe lower leg pain after a fall in a touch football game. Following routine triage, which of the following is the appropriate next step in assessment and treatment? A. Apply heat to the painful area. B. Apply an elastic bandage to the leg. C. X-ray the leg. D. Give pain medication. 11. A nurse caring for several patients on the cardiac unit is told that one is scheduled for implantation of an automatic internal cardioverter-defibrillator. Which of the following patients is most likely to have this procedure? A. A patient admitted for myocardial infarction without cardiac muscle damage. B. A post-operative coronary bypass patient, recovering on schedule. C. A patient with a history of ventricular tachycardia and syncopal episodes. D. A patient with a history of atrial tachycardia and fatigue. 12. A patient is scheduled for a magnetic resonance imaging (MRI) scan for suspected lung cancer. Which of the following is a contraindication to the study for this patient? A. The patient is allergic to shellfish. B. The patient has a pacemaker. C. The patient suffers from claustrophobia. D. The patient takes antipsychotic medication. 13. A nurse calls a physician with the concern that a patient has developed a pulmonary embolism. Which of the following symptoms has the nurse most likely observed? A. The patient is somnolent with decreased response to the family. B. The patient suddenly complains of chest pain and shortness of breath. C. The patient has developed a wet cough and the nurse hears crackles on auscultation of the lungs. D. The patient has a fever, chills, and loss of appetite. 14. A patient comes to the emergency department with abdominal pain. Work-up reveals the presence of a rapidly enlarging abdominal aortic aneurysm. Which of the following actions should the nurse expect? A. The patient will be admitted to the medicine unit for observation and medication. B. The patient will be admitted to the day surgery unit for sclerotherapy. C. The patient will be admitted to the surgical unit and resection will be scheduled. D. The patient will be discharged home to follow-up with his cardiologist in 24 hours. 15. A patient with leukemia is receiving chemotherapy that is known to depress bone marrow. A CBC (complete blood count) reveals a platelet count of 25,000/microliter. Which of the following actions related specifically to the platelet count should be included on the nursing care plan? A. Monitor for fever every 4 hours. B. Require visitors to wear respiratory masks and protective clothing. C. Consider transfusion of packed red blood cells. D. Check for signs of bleeding, including examination of urine and stool for blood. 16. A patient is undergoing the induction stage of treatment for leukemia. The nurse teaches family members about infectious precautions. Which of the following statements by family members indicates that the family needs more education? A. We will bring in books and magazines for entertainment. B. We will bring in personal care items for comfort. C. We will bring in fresh flowers to brighten the room. D. We will bring in family pictures and get well cards. 17. A nurse is caring for a patient with acute lymphoblastic leukemia (ALL). Which of the following is the most likely age range of the patient? A. 3-10 years. B. 25-35 years. C. 45-55 years. D. over 60 years. 18. A patient is admitted to the oncology unit for diagnosis of suspected Hodgkin’s disease. Which of the following symptoms is typical of Hodgkin’s disease? A. Painful cervical lymph nodes. B. Night sweats and fatigue. C. Nausea and vomiting. D. Weight gain. 19. The Hodgkin’s disease patient described in the question above undergoes a lymph node biopsy for definitive diagnosis. If the diagnosis of Hodgkin’s disease were correct, which of the following cells would the pathologist expect to find? A. Reed-Sternberg cells. B. Lymphoblastic cells. C. Gaucher’s cells. D. Rieder’s cells 20. A patient is about to undergo bone marrow aspiration and biopsy and expresses fear and anxiety about the procedure. Which of the following is the most effective nursing response? A. Warn the patient to stay very still because the smallest movement will increase her pain. B. Encourage the family to stay in the room for the procedure. C. Stay with the patient and focus on slow, deep breathing for relaxation. D. Delay the procedure to allow the patient to deal with her feelings. NCLEX Practice Exam 11 (20 Questions) 1. A mother complains to the clinic nurse that her 2 ½-year-old son is not yet toilet trained. She is particularly concerned that, although he reliably uses the potty seat for bowel movements, he isn’t able to hold his urine for long periods. Which of the following statements by the nurse is correct? A. The child should have been trained by age 2 and may have a psychological problem that is responsible for his “accidents.” B. Bladder control is usually achieved before bowel control, and the child should be required to sit on the potty seat until he passes urine. C. Bowel control is usually achieved before bladder control, and the average age for completion of toilet training varies widely from 24 to 36 months. D. The child should be told “no” each time he wets so that he learns the behavior is unacceptable. 2. The mother of a 14-month-old child reports to the nurse that her child will not fall asleep at night without a bottle of milk in the crib and often wakes during the night asking for another. Which of the following instructions by the nurse is correct? A. Allow the child to have the bottle at bedtime, but withhold the one later in the night. B. Put juice in the bottle instead of milk. C. Give only a bottle of water at bedtime. D. Do not allow bottles in the crib. 3. Which of the following actions is NOT appropriate in the care of a 2-month-old infant? A. Place the infant on her back for naps and bedtime. B. Allow the infant to cry for 5 minutes before responding if she wakes during the night as she may fall back asleep. C. Talk to the infant frequently and make eye contact to encourage language development. D. Wait until at least 4 months to add infant cereals and strained fruits to the diet. 4. An older patient asks a nurse to recommend strategies to prevent constipation. Which of the following suggestions would be helpful? Note: More than one answer may be correct. A. Get moderate exercise for at least 30 minutes each day. B. Drink 6-8 glasses of water each day. C. Eat a diet high in fiber. D. Take a mild laxative if you don’t have a bowel movement every day. 5. A child is admitted to the hospital with suspected rheumatic fever. Which of the following observations is NOT confirming of the diagnosis? A. A reddened rash visible over the trunk and extremities. B. A history of sore throat that was self-limited in the past month. C. A negative antistreptolysin O titer. D. An unexplained fever. 6. A patient with a history of congestive heart failure arrives at the clinic complaining of dyspnea. Which of the following actions is the first the nurse should perform? A. Ask the patient to lie down on the exam table. B. Draw blood for chemistry panel and arterial blood gas (ABG). C. Send the patient for a chest x-ray. D. Check blood pressure. 7. A clinic patient has recently been prescribed nitroglycerin for treatment of angina. He calls the nurse complaining of frequent headaches. Which of the following responses to the patient is correct? A. “Stop taking the nitroglycerin and see if the headaches improve.” B. “Go to the emergency department to be checked because nitroglycerin can cause bleeding in the brain.” C. “Headaches are a frequent side effect of nitroglycerine because it causes vasodilation.” D. “The headaches are unlikely to be related to the nitroglycerin, so you should see your doctor for further investigation.” 8. A patient received surgery and chemotherapy for colon cancer, completing therapy 3 months previously, and she is now in remission. At a follow-up appointment, she complains of fatigue following activity and difficulty with concentration at her weekly bridge games. Which of the following explanations could account for her symptoms? A. The symptoms may be the result of anemia caused by chemotherapy. B. The patient may be immunosuppressed. C. The patient may be depressed. D. The patient may be dehydrated. 9. A clinic patient has a hemoglobin concentration of 10.8 g/dL and reports sticking to a strict vegetarian diet. Which of the follow nutritional advice is appropriate? A. The diet is providing adequate sources of iron and requires no changes. B. The patient should add meat to her diet; a vegetarian diet is not advised. C. The patient should use iron cookware to prepare foods, such as dark green, leafy vegetables and legumes, which are high in iron. D. A cup of coffee or tea should be added to every meal. 10. A hospitalized patient is receiving packed red blood cells (PRBCs) for treatment of severe anemia. Which of the following is the most accurate statement? A. Transfusion reaction is most likely immediately after the infusion is completed. B. PRBCs are best infused slowly through a 20g. IV catheter. C. PRBCs should be flushed with a 5% dextrose solution. D. A nurse should remain in the room during the first 15 minutes of infusion. 11. Emergency department triage is an important nursing function. A nurse working the evening shift is presented with four patients at the same time. Which of the following patients should be assigned the highest priority? A. A patient with low-grade fever, headache, and myalgias for the past 72 hours. B. A patient who is unable to bear weight on the left foot, with swelling and bruising following a running accident. C. A patient with abdominal and chest pain following a large, spicy meal. D. A child with a one-inch bleeding laceration on the chin but otherwise well after falling while jumping on his bed. 12. A patient is admitted to the hospital with a calcium level of 6.0 mg/dL. Which of the following symptoms would you NOT expect to see in this patient? A. Numbness in hands and feet. B. Muscle cramping. C. Hypoactive bowel sounds. D. Positive Chvostek’s sign. 13. A nurse cares for a patient who has a nasogastric tube attached to low suction because of a suspected bowel obstruction. Which of the following arterial blood gas results might be expected in this patient? A. pH 7.52, PCO2 54 mmHg. B. pH 7.42, PCO2 40 mmHg. C. pH 7.25, PCO2 25 mmHg. D. pH 7.38, PCO2 36 mmHg. 14. A patient is admitted to the hospital for routine elective surgery. Included in the list of current medications is Coumadin (warfarin) at a high dose. Concerned about the possible effects of the drug, particularly in a patient scheduled for surgery, the nurse anticipates which of the following actions? A. Draw a blood sample for prothrombin (PT) and international normalized ratio (INR) level. B. Administer vitamin K. C. Draw a blood sample for type and crossmatch and request blood from the blood bank. D. Cancel the surgery after the patient reports stopping the Coumadin one week previously. 15. The follow lab results are received for a patient. Which of the following results are abnormal? Note: More than one answer may be correct. A. Hemoglobin 10.4 g/dL. B. Total cholesterol 340 mg/dL. C. Total serum protein 7.0 g/dL. D. Glycosylated hemoglobin A1C 5.4%. 16. A nurse is assigned to the pediatric rheumatology clinic and is assessing a child who has just been diagnosed with juvenile idiopathic arthritis. Which of the following statements about the disease is most accurate? A. The child has a poor chance of recovery without joint deformity. B. Most children progress to adult rheumatoid arthritis. C. Nonsteroidal anti-inflammatory drugs are the first choice in treatment. D. Physical activity should be minimized. 17. A child is admitted to the hospital several days after stepping on a sharp object that punctured her athletic shoe and entered the flesh of her foot. The physician is concerned about osteomyelitis and has ordered parenteral antibiotics. Which of the following actions is done immediately before the antibiotic is started? A. The admission orders are written. B. A blood culture is drawn. C. A complete blood count with differential is drawn. D. The parents arrive. 18. A two-year-old child has sustained an injury to the leg and refuses to walk. The nurse in the emergency department documents swelling of the lower affected leg. Which of the following does the nurse suspect is the cause of the child’s symptoms? A. Possible fracture of the tibia. B. Bruising of the gastrocnemius muscle. C. Possible fracture of the radius. D. No anatomic injury, the child wants his mother to carry him. 19. A toddler has recently been diagnosed with cerebral palsy. Which of the following information should the nurse provide to the parents? Note: More than one answer may be correct. A. Regular developmental screening is important to avoid secondary developmental delays. B. Cerebral palsy is caused by injury to the upper motor neurons and results in motor dysfunction, as well as possible ocular and speech difficulties. C. Developmental milestones may be slightly delayed but usually will require no additional intervention. D. Parent support groups are helpful for sharing strategies and managing health care issues. 20. A child has recently been diagnosed with Duchenne’s muscular dystrophy. The parents are receiving genetic counseling prior to planning another pregnancy. Which of the following statements includes the most accurate information? A. Duchenne’s is an X-linked recessive disorder, so daughters have a 50% chance of being carriers and sons a 50% chance of developing the disease. B. Duchenne’s is an X-linked recessive disorder, so both daughters and sons have a 50% chance of developing the disease. C. Each child has a 1 in 4 (25%) chance of developing the disorder. D. Sons only have a 1 in 4 (25%) chance of developing the disorder. [Show More]

Last updated: 1 year ago

Preview 1 out of 187 pages

Reviews( 2 )

user-profile-pic


by Jehus cristobal · 3 years ago

Hdududduduie

Thank you for the good review. by Kirsch. 3 years ago

user-profile-pic


by Kirsch · 3 years ago

Thank you for the good review.

$13.50

Add to cart

Instant download

Can't find what you want? Try our AI powered Search

OR

GET ASSIGNMENT HELP
265
1

Document information


Connected school, study & course


About the document


Uploaded On

Jun 05, 2020

Number of pages

187

Written in

Seller


seller-icon
Kirsch

Member since 4 years

899 Documents Sold


Additional information

This document has been written for:

Uploaded

Jun 05, 2020

Downloads

 1

Views

 265

Document Keyword Tags

Recommended For You


$13.50
What is Browsegrades

In Browsegrades, a student can earn by offering help to other student. Students can help other students with materials by upploading their notes and earn money.

We are here to help

We're available through e-mail, Twitter, Facebook, and live chat.
 FAQ
 Questions? Leave a message!

Follow us on
 Twitter

Copyright © Browsegrades · High quality services·